Работы под наведенным напряжением: XXXVIII. Охрана труда при выполнении работ на воздушных линиях электропередачи / КонсультантПлюс

Содержание

Работы под наведенным напряжением | Вопросы и ответы по правилам безопасной эксплуатации электроустановок

Страница 25 из 36

Раздел 15, Глава 3
РАБОТЫ НА ТОКОВЕДУЩИХ ЧАСТЯХ ЭЛЕКТРОУСТАНОВОК И ВЛ ЭЛЕКТРОПЕРЕДАЧИ ПОД НАВЕДЕННЫМ НАПРЯЖЕНИЕМ

Вопрос 1
п.16.3.1 Кто допускается к работам на токоведущих частях электроустановок под наведенным напряжением?
1(*) Работники, прошедшие специальное обучение методам безопасного выполнения таких работ.
2(*) Работники, прошедшие проверку знаний.
3(*) Работники, имеющие запись в удостоверении о предоставлении права проведения таких работ.

Вопрос 2
п.16.3.1. Члены бригады, выполняющие работы под наведенным напряжением, должны иметь группу по электробезопасности…
1(*) 3 .

Вопрос 3
пп.16.3.1,16.3.2. Укажите требования к организации работ под наведенным напряжением.
1(*) Работники, обслуживающие электроустановки и ВЛ, должны знать перечень линий электропередачи и линейного оборудования, которые после отключения находятся под наведенным напряжением.

2(*) К работам должны допускаться члены бригады с группой по электробезопасности 3, прошедшие обучение и проверку знаний и имеющие отметку в удостоверении о предоставлении права выполнять такие работы.
3 Водители машин и механизмов должны иметь группу по электробезопасности 3.

Вопрос 4
п.16.3.3. Как определяются зоны сильного действия ВЛ из числа находящихся под наведенным напряжением?
1 Путем расчетов и измерений при рабочем токе перед началом работ на ВЛ.
2(*) Путем расчетов и измерений при наибольшем токе ВЛ.
3(*) При отключении и заземлении ВЛ по концам и на месте работ.

Вопрос 5
п.16.3.3. Когда следует повторно проводить определение зон сильного действия наведенного напряжения на ВЛ?
1 Через каждые 3 года.
2(*) При изменении схемы или режима электросети.
3 Пред началом работы на данной ВЛ.

Вопрос 6
п.16.3.4. Где должна быть заземлена ВЛ для выполнения работ в зоне слабого действия наведенного напряжения?

1 На месте работ.
2(*) В РУ электростанций и подстанций.

Вопрос 7
п.16.3.5. Для выполнения работ в зоне сильного действия наведенного напряжения ВЛ должна быть заземлена:
1(*) На месте работ.
2 В РУ электростанций и подстанций.

Вопрос 8
пп.16.3.4,16.3.5. В каких случаях ВЛ под наведенным напряжением заземляется в РУ электростанций и подстанций?
1(*) При выполнении работ на ВЛ в зоне слабого действия наведенного напряжения.
2(*) На участке совместного прохождения ВЛ вблизи РУ электростанций (подстанций), но не далее 2 км от них — заземление в конечных РУ.
3(*) На участке одиночного прохождения, примыкающего к РУ электростанций и подстанций — заземление в РУ, примыкающем к участку работ.

Вопрос 9
п.16.3.6. Какие дополнительные меры безопасности, препятствующие ошибочному или самопроизвольному включению коммутационных аппаратов, надо принять при работах на ВЛ под наведенным напряжением сильного действия, заземленных только на месте работ?

1(*) Разобрать схему выключателя с двух сторон.
2(*) На линейном разъединителе включить заземляющие ножи в сторону выключателя.
3(*) При наличии обходной системы шин ее заземлить.

Вопрос 10
п.16.3.7. Когда используется одиночный стержневой заземлитель при работах под наведенным напряжением?
1(*) При выполнении работ в зоне слабого действия наведенного напряжения.
2 В случае выполнения работ в зоне сильного действия наведенного напряжения.

Вопрос 11
п.16.3.7. Укажите методы заземления токоведущих частей электроустановки, находящейся под наведенным напряжением.
1(*) Заземления устанавливаются на каждом рабочем месте с присоединением к контуру заземления опоры или к заземляющему устройству подстанции.
2 Заземления устанавливаются на каждом рабочем месте с присоединением к стержневому заземлителю.
3(*) Заземление устанавливается на каждом рабочем месте с присоединением к групповому заземлителю.

Вопрос 12
п.16.3.8. Что запрещается делать с момента заземления провода при наведенном напряжении?
1(*) Касаться с земли заземляющих проводников, монтажных канатов, машин и механизмов без средств защиты.
2(*) Заходить в кабину механизма и выходить из нее.

Вопрос 13
п.16.3.9. При выполнении работ на участке совместного прохождения ВЛ при наведенном напряжении сильного действия базовое заземление следует устанавливать не далее … км от рабочего места.
1(*) 1 .

Вопрос 14
п.16.3.9. Где допускается установка базового заземления при работах на опоре участка одиночного прохождения ВЛ в зоне сильного действия наведенного напряжения?
1 На опоре, на которой проводятся работы.
2(*) В любом месте участка одиночного прохождения ВЛ.

Вопрос 15
п.16.3.9. Укажите дополнительные организационные мероприятия при установке базового заземления.
1(*) Базовое заземление устанавливается по отдельному наряду.

2 Задание на установку базового заземления выдаётся оперативным работник.
3(*) Задание на установку и снятие базового заземления выдается оперативным работником, который выдаёт разрешение на подготовку рабочих мест.

Вопрос 16
п.16.3.10. Каковы правила установки базового заземления?
1(*) Установка и снятие базового заземления выполняются с заземлением всех фаз на контур заземления опоры или на групповой заземлитель.
2(*) Допускается в зависимости от местных условий без заземления ВЛ в РУ электростанций (подстанций) с записью в оперативном журнале.
3(*) Допускается с временным заземлением в РУ в зависимости от местных условий.
4(*) Установку и снятие заземления должны выполнять два члена бригады с группами по электробезопасности 3 и 4 под надзором руководителя работ.
5(*) В строке «Отдельные указания» наряда записываются уровни наведенных напряжений до установки базового заземления и после подготовки рабочих мест.

6(*) Базовое заземление устанавливается до начала подготовительных работ и снимается после полного окончания работ.

Вопрос 17
п.16.3.11. В случае совместного прохождения ВЛ в зоне сильного действия наведенного напряжения допускается одновременная работа нескольких бригад на участке длиной до … км.
1(*) 2 .

Вопрос 18
п.16.3.11. В зоне сильного действия наведенного напряжения бригадам необходимо работать на участке длиной 5 км. Каковы условия выполнения этих работ?
1(*) Разделить ВЛ на электрически не связанные между собой участки.
2(*) Установить на каждом электрически не связанном с другими участке базовое заземление.

Вопрос 19
п.16.3.12. Где нужно установить базовое заземление при работах на ВЛ, проходящих на территории разных предприятий?
1(*) На каждом участке работ.
2 На крайних участках работ.

Вопрос 20
п.16.3.13. Укажите последовательность указанных работ при монтаже проводов на ВЛ под наведенным напряжением.

1(*) Монтаж такелажной схемы на земле.
2(*) Заземление такелажной схемы на общий заземлитель.
3(*) Заземление провода.

Вопрос 21
п.16.3.14. Как должны проводиться работы, связанные с прикосновением к опущенному до земли проводу, на ВЛ под наведенным напряжением?
1(*) С использованием электрозащитных средств.
2(*) С металлической площадки, соединенной с проводом для выравнивания потенциалов.
3 Без применения электрозащитных средств и металлической площадки при условии заземления провода в непосредственной близости от каждого места прикосновения.
4(*) Входить на металлическую площадку и сходить с неё в диэлектрической обуви.

Вопрос 22
п.16.3.15. Перед разрезанием провода ВЛ, находящейся под наведенным напряжением, его следует заземлить с двух сторон разреза:
1(*) На контур заземления опоры.
2 На два индивидуальных заземлителя.
3(*) На общий групповой заземлитель.

Вопрос 23
п.16.3.16. На какое расстояние запрещается приближаться к заземлителю без диэлектрической обуви? Менее … м.
1(*) 3 .

Вопрос 24
п.16.3.17. Монтаж и замена провода под наведенным напряжением и все работы, связанные с прикосновением к роводу, машинам, механизмам, следует выполнять при условии…
1(*) Заземления их на месте работ.
2(*) Использования средств электрозащиты.
3 Заземления в РУ станций и подстанций.

Вопрос 25
п.16.3.18. Подъем и опускание провода на ВЛ под наведенным напряжением должны проводиться…
1(*) С заземлением провода на каждой опоре, где осуществляется монтаж, при условии, что длина участка не более 2 км.
2 С заземлением провода на каждой опоре, где ведется монтаж, на участке длиной не более 3 км.

Вопрос 26
п.16.3.19. Укажите требования к перекладке проводов из раскаточных роликов в зажимы и обратно на ВЛ под наведенным напряжением.
1 Перекладка должна проводиться в направлении, обратном направлению раскатки.

2(*) Провод должен перекладываться после его заземления на месте работ или на соседней опоре.

Вопрос 27
п.16.3.20. До начала работ по соединению проводов в петлях анкерных опор ВЛ 110 кВ и выше петли следует закрепить:
1(*) За провода.
2(*) За натяжные изолирующие подвески.
3 Настоящие Правила это не регламентируют.

Вопрос 28
п.16.3.21. Когда можно начинать соединять провода в петлях анкерных опор?
1(*) После полного прекращения работ в смежных анкерных пролетах.
2(*) После снятия всех заземлений в смежных анкерных пролетах.
3 Линия должна быть заземлена на смежных опорах.
4(*) Линия должна быть заземлена на анкерной опоре, где проводятся работы, с заземлением концов соединяемых проводов на контур заземления опоры.

Вопрос 29
п.16.3.22. Каковы дополнительные меры безопасности (кроме включения заземляющих ножей в сторону линии) при работах на оборудовании электростанций и подстанций, находящемся под наведенным напряжением?

1(*) Установка на спуски проводов со стороны ВЛ по одному переносному заземлению.
2(*) Включение заземляющих ножей на обходном разъединителе в сторону ВЛ, если на нем не проводятся работы.

Вопрос 30
п.16.3.22. При каком условии на подстанциях следует устанавливать и снимать переносное заземление в сторону линии при наведенном напряжении?
1(*) При включении заземляющих ножей на обходной системе шин.
2(*) При включении заземляющих ножей на линейном разъединителе.
3 При включении заземляющих ножей на разъединителе в сторону выключателя.

Вопрос 31
п.16.3.23. Каковы требования к работам, выполняемым с телескопических вышек и гидроподъемников в электроустановках под наведенным напряжением?
1(*) Рабочую площадку механизма следует соединить с заземленным на месте работ проводом-перемычкой из гибкого медного провода при помощи специальной штанги.

2(*) Механизм должен быть заземлен на общий с проводом заземлитель.
3 Сечение перемычки и заземляющего проводника должно быть не менее 50 кв.мм.

Вопрос 32
п.16.3.23. Медный проводник с какой площадью сечения следует применять для заземления телескопической вышки при работах под наведенным напряжением? Не менее … кв.мм?
1(*) 25 .

Работы на ВЛ под наведенным напряжением; на одной отключенной цепи многоцепной ВЛ

Работы на ВЛ под наведенным напряжением; на одной отключенной цепи многоцепной ВЛ

4.15.43. Персонал, обслуживающий ВЛ, должен иметь перечень линий, которые после отключения находятся под наведенным напряжением, ознакомлен с этим перечнем, значениями наводимого напряжения. Наличие наведенного напряжения на ВЛ должно быть записано в строке «Отдельные указания» наряда.

4.15.44. В случаях наличия на отключенных ВЛ и ВЛС наведенного напряжения перед соединением или разрывом электрически связанных участков (проводов, тросов) необходимо выровнять потенциалы этих участков. Уравнивание потенциалов осуществляется путем соединения проводником этих участков или установкой заземлений по обе стороны разрыва (предполагаемого разрыва) с присоединением к одному заземлителю (заземляющему устройству).

4.15.45. На ВЛ под наведенным напряжением работы с земли, связанные с прикосновением к проводу, опущенному с опоры вплоть до земли, должны выполняться с использованием электрозащитных средств (диэлектрические перчатки, штанги) или с металлической площадки, соединенной для выравнивания потенциалов проводником с этим проводом. Работы с земли без применения электрозащитных средств и металлической площадки допускаются при условии заземления провода в непосредственной близости к каждому месту прикосновения.

4.15.46. Применяемые при монтаже проводов на ВЛ под наведенным напряжением стальные тяговые канаты сначала необходимо закреплять на тяговом механизме и для выравнивания потенциалов заземлять на тот же заземлитель, что и провод. Только после этого разрешается прикреплять канат к проводу. Разъединять провод и тяговый канат можно только после выравнивания их потенциалов, т. е. после соединения каждого из них с общим заземлителем.

4.15.47. При монтажных работах на ВЛ под наведенным напряжением (подъем, визирование, натяжка, перекладка проводов из раскаточных роликов в зажимы) провод должен быть заземлен на анкерной опоре, от которой ведется раскатка, на конечной анкерной опоре, через которую проводится натяжка, и на каждой промежуточной опоре, на которую поднимается провод.

4.15.48. По окончании работы на промежуточной опоре заземление с провода на этой опоре может быть снято. В случае возобновления работы на промежуточной опоре, связанной с прикосновением к проводу, провод должен быть вновь заземлен на той же опоре.

4.15.49. На ВЛ под наведенным напряжением перекладку проводов из раскаточных роликов в поддерживающие зажимы следует проводить в направлении, обратном направлению раскатки. До начала перекладки необходимо, оставив заземленными провода на анкерной опоре, в сторону которой будет проводиться перекладка, снять заземление с проводов на анкерной опоре, от которой начинается перекладка.

4.15.50. При монтаже проводов на ВЛ под наведенным напряжением заземления с них можно снимать только после перекладки провода в поддерживающие зажимы и окончания работ на данной опоре.

4.15.51. Во время перекладки проводов в зажимы смежный анкерный пролет, в котором перекладка уже закончена, следует рассматривать как находящийся под наведенным напряжением. Выполнять на нем работы, связанные с прикосновением к проводам, разрешается только после заземления их на рабочем месте.

4.15.52. Из числа ВЛ под наведенным напряжением организациям необходимо определить измерениями линии, при отключении и заземлении которых по концам (в РУ) на заземленных проводах остается потенциал наведенного напряжения выше 25 В при наибольшем рабочем токе действующей ВЛ.

Все виды работ на этих ВЛ, связанные с прикосновением к проводу без применения основных электрозащитных средств, должны выполняться по технологическим картам или ППР, в которых должно быть указано размещение заземлений исходя из требований обеспечения на рабочих местах потенциала наведенного напряжения не выше 25 В.

4.15.53. Если на отключенной ВЛ (цепи), находящейся под наведенным напряжением, не удается снизить это напряжение до 25 В, необходимо работать с заземлением проводов только на одной опоре или на двух смежных. При этом заземлять ВЛ (цепь) в РУ не допускается. Допускается работа бригады только с опор, на которых установлены заземления, или на проводе в пролете между ними.

4.15.54. При необходимости работы в двух и более пролетах (участках) ВЛ (цепь) должна быть разделена на электрически не связанные участки посредством разъединения петель на анкерных опорах. На каждом из таких участков у мест установки заземлений может работать лишь одна бригада.

4.15.55. На отключенной цепи многоцепной ВЛ с расположением цепей одна над другой можно работать только при условии, что эта цепь подвешена ниже цепей, находящихся под напряжением. Не допускается заменять и регулировать провода отключенной цепи.

4.15.56. При работе на одной отключенной цепи многоцепной ВЛ с горизонтальным расположением цепей на стойках должны быть вывешены красные флажки со стороны цепей, оставшихся под напряжением. Флажки вывешивают на высоте 2–3 м от земли производитель работ с членом бригады, имеющим группу III.

4.15.57. Подниматься на опору со стороны цепи, находящейся под напряжением, и переходить на участки траверс, поддерживающих эту цепь, не допускается. Если опора имеет степ-болты, подниматься по ним разрешается независимо от того, под какой цепью они расположены. При расположении степ-болтов со стороны цепей, оставшихся под напряжением, подниматься на опору следует под наблюдением находящегося на земле производителя работ или члена бригады, имеющего группу III.

4.15.58. При работе с опор на проводах отключенной цепи многоцепной ВЛ, остальные цепи которой находятся под напряжением, заземление необходимо устанавливать на каждой опоре, на которой ведутся работы.

Данный текст является ознакомительным фрагментом.

Продолжение на ЛитРес

Работы на воздушных линиях под наведенным напряжением.

Раздел VI

ВОЗДУШНЫЕ ЛИНИИ ЭЛЕКТРОПЕРЕДАЧИ

Глава 41

РАБОТЫ НА ВЛ ПОД НАВЕДЕННЫМ НАПРЯЖЕНИЕМ, НА ОДНОЙ ОТКЛЮЧЕННОЙ ЦЕПИ МНОГОЦЕПНОЙ ВЛ

367. Работники, обслуживающие ВЛ, должны иметь перечень линий, которые после отключения находятся под наведенным напряжением, ознакомлены с этим перечнем и величинами наводимого напряжения. Наличие наведенного напряжения на ВЛ должно быть записано в строке «Категория работ» наряда.

368. На ВЛ под наведенным напряжением работы с земли с прикосновением к проводу, опущенному с опоры вплоть до земли, должны выполняться с применением электрозащитных средств (электроизолирующие перчатки, штанги) или с металлической площадки, соединенной с этим проводом для выравнивания потенциалов проводников. Работы с земли без применения электрозащитных средств и металлической площадки допускаются при условии заземления провода в непосредственной близости к каждому месту прикосновения.

369. Применяемые при монтаже проводов на ВЛ под наведенным напряжением стальные тяговые канаты сначала необходимо закреплять на тяговом механизме и для выравнивания потенциалов заземлять на тот же заземлитель, что и провод. После этого разрешается прикреплять канат к проводу. Разъединять провод и тяговый канат можно после выравнивания их потенциалов, то есть после соединения каждого из них с общим заземлителем.

370. При монтажных работах на ВЛ под наведенным напряжением (подъем, визировка, натяжка, перекладка проводов из раскаточных роликов в зажимы) провод должен быть заземлен на анкерной опоре, от которой ведется раскатка, на конечной анкерной опоре, через которую проводится натяжка, и на каждой промежуточной опоре, на которую поднимается провод.

371. На ВЛ или ВЛС перед соединением или разрывом электрически связанных участков (проводов, тросов) необходимо выровнять потенциалы этих участков. Выравнивание потенциалов осуществляется путем соединения проводником этих участков линии или установки заземлений по обе стороны разрыва (предполагаемого разрыва) с присоединением их к одному заземлителю (заземляющему устройству).

372. После окончания работ на промежуточной опоре заземление провода на этой опоре может быть снято. В случае возобновления работы на промежуточной опоре, связанной с прикосновением к проводу, провод должен быть вновь заземлен на этой опоре.

373. На ВЛ под наведенным напряжением перекладку проводов из раскаточных роликов в поддерживающие зажимы следует проводить в направлении, обратном направлению раскатки. До начала перекладки необходимо, оставив заземленными провода на анкерной опоре, в сторону которой будет проводиться перекладка, снять заземление с проводов на анкерной опоре, от которой начинается перекладка.

374. При монтаже проводов на ВЛ под наведенным напряжением заземления с них можно снимать только после перекладки провода в поддерживающие зажимы и окончания работ на данной опоре.

375. Во время перекладки проводов в зажимы смежный анкерный пролет, в котором перекладка уже закончена, следует считать находящимся под наведенным напряжением. Выполнять на нем работы с прикосновением к проводам разрешается только после заземления их на рабочем месте.

376. Организациям, выполняющим работы на ВЛ и ВЛС под наведенным напряжением, необходимо определить линии, при отключении и заземлении которых по концам (в РУ) на заземленных проводах наведенное напряжение превышает 42В при наибольшем рабочем токе действующей ВЛ. К таким линиям относятся ВЛ, ВЛС и контактные сети электрифицированной железной дороги, которые по всей длине или на отдельных участках общей длиной не менее 2 км проходят на расстоянии от оси другой ВЛ напряжением 110кВ и выше:

для ВЛ напряжением 110кВ — 100м;

для ВЛ напряжением 220кВ — 150м;

для ВЛ напряжением 330кВ — 200м;

для ВЛ напряжением 750кВ — 250м и менее.

Все виды работ, связанные с прикосновением к проводам этих ВЛ без применения основных электрозащитных средств, должны проводиться по технологическим картам или ППР. В них должно быть указано размещение заземлений исходя из требований обеспечения наведенного напряжения не выше 42В на рабочих местах.

377. Если на отключенной ВЛ (цепи), находящейся под наведенным напряжением, не удается уменьшить это напряжение до 42 В, необходимо выполнять работы с заземлением проводов только по одной опоре или двум смежным опорам. При этом заземлять ВЛ (цепь) в РУ запрещается. Допускается работа бригады только на опорах, на которых установлены заземления, и в пролете между ними.

При необходимости проведения работы в двух и более пролетах (участках) ВЛ цепь должна быть разделена на электрически не связанные участки путем разъединения петель на анкерных опорах. На каждом из таких участков у мест установки заземлений может работать только одна бригада.

378. На отключенной цепи многоцепной ВЛ с расположением цепей одна над другой допускается работать только при подвеске этой цепи ниже цепей, находящихся под напряжением. Запрещается заменять и регулировать провода отключенной цепи.

379. При работе на одной отключенной цепи многоцепной ВЛ с горизонтальным расположением цепей на стойках опор должны быть вывешены красные флажки со стороны цепей, оставшихся под напряжением. Флажки вывешивают на высоте 2 — 3м от земли производитель работ с членом бригады, имеющим группу по электробезопасности III.

380. Подниматься на опору ВЛ со стороны цепи, находящейся под напряжением, и переходить на участки траверс, поддерживающих эту цепь, запрещается. Если опора имеет степ-болты, подниматься по ним разрешается независимо от того, под какой цепью они расположены. При расположении степ-болтов со стороны цепей, находящихся под напряжением, подниматься на опору следует под наблюдением производителя работ или члена бригады, имеющего группу по электробезопасности III и находящегося на земле.

381. При работе с опор на проводах отключенной цепи многоцепной ВЛ, остальные цепи которой находятся под напряжением, заземление необходимо устанавливать на каждой опоре, на которой ведутся работы.

XVI. Особенности производства переключений при выводе в ремонт и вводе в работу ЛЭП под наведенным напряжением [ПРАВИЛА ПЕРЕКЛЮЧЕНИЙ В ЭЛЕКТРОУСТАНОВКАХ] — последняя редакция

XVI. Особенности производства переключений при выводе
в ремонт и вводе в работу ЛЭП под наведенным напряжением

188. Собственники или иные законные владельцы ВЛ, КВЛ обязаны обеспечивать регулярное проведение замеров уровней наведенного напряжения на всех ВЛ и воздушных участках КВЛ, находящихся в их эксплуатационном обслуживании.

Информация о факте отнесения ВЛ и КВЛ к ЛЭП под наведенным напряжением должна быть предоставлена собственником или иным законным владельцем ЛЭП в ДЦ или ЦУС, в чьем диспетчерском или технологическом управлении находится соответствующая ЛЭП.

189. Вывод в ремонт ЛЭП под наведенным напряжением должен осуществляться в соответствии с требованиями правил по охране труда при эксплуатации электроустановок с использованием одной из следующих схем заземления:

с включением заземляющих разъединителей в сторону ЛЭП во всех распределительных устройствах, к которым подключена ЛЭП, и установкой заземления на рабочем месте;

без включения заземляющих разъединителей в сторону ЛЭП во всех распределительных устройствах, к которым подключена ЛЭП, с установкой заземления на рабочем месте.

190. В случае применения при выводе в ремонт ЛЭП напряжением 35 кВ и выше с отпайками схемы заземления, указанной в абзаце втором пункта 189 Правил, допускается ЛЭП не заземлять на отпаечных подстанциях, при условии, что ЛЭП заземлена с двух сторон, а на отпаечных подстанциях установлены заземления за отключенными линейными разъединителями.

191. В случае применения при выводе в ремонт ЛЭП схемы заземления, указанной в абзаце третьем пункта 189 Правил, и определения организацией, эксплуатирующей ЛЭП, необходимости включения заземляющих разъединителей в сторону ЛЭП в одном из распределительных устройств, к которым подключена ЛЭП, на время установки (снятия) заземления на рабочем месте, при выводе ЛЭП в ремонт должна соблюдаться следующая последовательность операций:

отключаются выключатели (отделители) ЛЭП в распределительных устройствах объектов электроэнергетики со всех сторон, откуда может быть подано напряжение;

отключаются линейные разъединители со всех сторон, откуда может быть подано напряжение (в том числе разъединители ОСШ и иные разъединители в зависимости от схемы присоединения ЛЭП к распределительному устройству), и проверяется их отключенное положение по месту установки;

принимаются меры, препятствующие подаче напряжения на место производства работ вследствие ошибочного или самопроизвольного включения коммутационных аппаратов;

вывешиваются запрещающие плакаты на приводах ручного и ключах дистанционного управления коммутационных аппаратов;

проверяется отсутствие напряжения на ЛЭП и включается заземляющий разъединитель в сторону ЛЭП только на одном объекте переключений. Объект переключений, на котором производится заземление ЛЭП, должен определяться собственником или иным законным владельцем ЛЭП по согласованию с ДЦ, ЦУС, НСО (для ЛЭП, находящихся в управлении) исходя из обеспечения безопасности и оптимальной последовательности производства переключений в электроустановках;

диспетчерский персонал ДЦ (оперативный персонал ЦУС, НСО), в диспетчерском (технологическом) управлении которого находится ЛЭП, сообщает о заземлении ЛЭП только на одном объекте переключений уполномоченному работнику, выдающему разрешение на подготовку рабочего места и допуск к работе на ЛЭП;

устанавливается переносное заземление на ЛЭП в месте производства работ в соответствии с требованиями правил по охране труда при эксплуатации электроустановок;

уполномоченный работник, выдающий разрешение на подготовку рабочего места и допуск к работе на ЛЭП, сообщает диспетчерскому персоналу ДЦ (оперативному персоналу ЦУС, НСО), в диспетчерском (технологическом) управлении которого находится ЛЭП, об установке переносного заземления на месте производства работ;

отключается заземляющий разъединитель в сторону ЛЭП, включенный в соответствии с абзацем шестым настоящего пункта;

диспетчерский персонал ДЦ (оперативный персонал ЦУС, НСО), в диспетчерском (технологическом) управлении которого находится ЛЭП, сообщает уполномоченному работнику, выдающему разрешение на подготовку рабочего места и допуск к работе на ЛЭП, об отключении заземляющего разъединителя в сторону ЛЭП;

уполномоченный работник, выдающий разрешение на подготовку рабочего места и допуск к работе на ЛЭП, выдает разрешение на подготовку рабочего места и допуск к производству работ на ЛЭП в соответствии с требованиями правил по охране труда при эксплуатации электроустановок;

при необходимости перевода работающей на ЛЭП бригады на другое рабочее место после сообщения об окончании работ на данном месте работ и удалении бригады выполняются операции согласно абзацам шестому и седьмому настоящего пункта, переносное заземление снимается с данного места работ и устанавливается на другом месте работ, выполняются операции согласно абзацам девятому — двенадцатому настоящего пункта. При перемещении бригады на другое рабочее место может быть изменен объект переключений, на котором включается заземляющий разъединитель в сторону ЛЭП.

192. В указанном в абзаце первом пункта 191 Правил случае при вводе ЛЭП в работу должна соблюдаться следующая последовательность операций:

уполномоченный работник, выдавший разрешение на подготовку рабочего места и допуск к работе на ЛЭП, сообщает диспетчерскому персоналу ДЦ (оперативному персоналу ЦУС, НСО), в диспетчерском (технологическом) управлении которого находится ЛЭП, об окончании работ на ЛЭП и необходимости включения заземляющего разъединителя на одном объекте переключений, определяемом в соответствии с абзацем шестым пункта 191 Правил, в сторону ЛЭП для обеспечения безопасности при снятии переносного заземления с ЛЭП на рабочем месте;

осуществляются проверка отсутствия напряжения на ЛЭП и включение заземляющего разъединителя в сторону ЛЭП;

диспетчерский персонал ДЦ (оперативный персонал ЦУС, НСО), в диспетчерском (технологическом) управлении которого находится ЛЭП, сообщает уполномоченному работнику, выдавшему разрешение на подготовку рабочего места и допуск к работам на ЛЭП, о включении заземляющего разъединителя в сторону ЛЭП на одном объекте переключений;

на ЛЭП в месте производства работ в соответствии с требованиями правил по охране труда при эксплуатации электроустановок осуществляется снятие переносного заземления;

уполномоченный работник, выдающий разрешение на подготовку рабочего места и допуск к работам на ЛЭП, сообщает диспетчерскому персоналу ДЦ (оперативному персоналу ЦУС, НСО), в диспетчерском (технологическом) управлении которого находится ЛЭП, о снятии переносного заземления с ЛЭП в месте производства работ.

Операции по выводу в ремонт и вводу в работу ЛЭП напряжением 500 кВ А-Б под наведенным напряжением при необходимости включения заземляющего разъединителя в сторону ЛЭП на время установки заземления на рабочем месте должны производиться с соблюдением вышеуказанных требований и последовательности, указанной в приложении N 4 к Правилам.

Работы на токоведущих частях электроустановок и воздушных линий электропередач под наведенным напряжением*

____________

* Требования этого раздела относятся к работам на проводах,

тросах и линейном оборудовании электростанций (подстанций).

6.1.53. К работам на токоведущих частях электроустановок под наведенным напряжением могут быть допущены лица, прошедшие специальное обучение методам безопасного выполнения таких работ, с проверкой знаний и записью в удостоверении о предоставлении права на их проведение.

Члены бригады (за исключением водителей машин и механизмов) должны иметь группу по электробезопасности не ниже III.

6.1.54. Работники, обслуживающие электроустановки и ВЛ, должны знать перечень ВЛ и линейного оборудования электростанций и подстанций, находящихся под наведенным напряжением после их отключения.

6.1.55. Из числа ВЛ, под наведенным напряжением, следует определять измерениями или расчетами линии, при отключении и заземлении которых по концам (в РУ) и на месте выполнения работ на заземлителе остается потенциал наведенного напряжения выше 42 В при наибольшем рабочем токе действующих ВЛ. В дальнейшем измерения или расчеты следует производить при смене режима, схемы сети.

6.1.56. При работах в зоне слабого действия наведенного напряжения линия должна быть заземлена в РУ электростанций и подстанций.

6.1.57. Работы в зоне сильного действия наведенного напряжения следует выполнять без заземления ВЛ в РУ электростанций и подстанций. Исключение составляют работы, выполняемые:

— на участке совместимого следования ВЛ вблизи РУ электростанций (подстанций), но не далее 2 км от них — в этих случаях ВЛ заземляется на конечных РУ;

— на участке одиночного следования, примыкающем к РУ электростанций (подстанций) — в этих случаях линию следует заземлять на примыкающем к участку работ РУ.

6.1.58. При выполнении работ на ВЛ в зоне сильного действия наведенного напряжения, когда эта линия не заземлена в РУ электростанции и подстанции, должны быть приняты дополнительные меры препятствующие ошибочному или самопроизвольному включению коммутационных аппаратов. Для этого дополнительно к требованиям пункта 4.2.4 настоящих Правил схема выключателя должна быть разобрана разъединителями с обеих сторон, а на линейном разъединителе должны быть включены заземляющие ножи в сторону выключателя. При наличии обходной системы шин она также должна быть заземлена.

6.1.59. Токоведущие части электроустановки, находящиеся под наведенным напряжением, должны быть заземлены на каждом рабочем месте с присоединением заземляющих проводников к контуру заземления опоры или к заземляющему устройству электростанции или подстанции. Разрешается использовать групповой заземлитель. Применение одиночного стержневого заземлителя допускается только при работах в зоне слабого действия наведенного напряжения.

6.1.60. С момента заземления провода заземлитель, заземляющие проводники, опоры и их элементы, монтажные канаты, машины и механизмы следует считать, находящимися под напряжением, и прикасаться к ним стоя на земле без применения электрозащитных средств (диэлектрических перчаток, обуви), а также входить в кабину механизма и выходить из нее — запрещается.

6.1.61. Работы в зоне сильного действия наведенного напряжения, выполняемые без заземления ВЛ в РУ электростанции и подстанции, должны производиться с установкой базового заземлителя на участке производства работ. При работе на участке совместимого следования ВЛ базовый заземлитель необходимо устанавливать не далее 1 км от рабочего места, а на участках одиночного прохождения линии — его можно размещать произвольно в пределах этого участка. Не допускается установка базового заземлителя на опоре, где выполняются работы.

6.1.62. Установка и снятие базового заземления выполняется с заземлением проводов всех фаз на контур заземления опоры, а в случае отсутствия такого контуру — на групповой заземлитель. В зависимости от местных условий допускается устанавливать и снимать базовое заземление без заземления ВЛ в РУ электростанции (подстанции) с записью в оперативном журнале или временным заземлением ВЛ в этих РУ. Установку и снятие базового заземления должен выполнять руководитель работ с двумя членами бригады с группами IV и III.

В строке «Отдельные указания» наряда необходимо указать расчетные уровни наведенного напряжения до установки базового заземления и после подготовки рабочего места.

Базовое заземление устанавливается перед началом подготовительных работ и снимается после полного окончания работ и снятия рабочих заземлений.

6.1.63. Работы в зоне сильного действия наведенного напряжения, при совместном следовании ВЛ, следует выполнять на одной опоре или двух смежных и пролете между ними. При совместном следовании ВЛ в зоне сильного действия наведенного напряжения допускается одновременное проведение работ несколькими бригадами, чтобы длина участка работ не превышала 2 км, при условии выполнения требований пункта 6.1.61 настоящих Правил. При необходимости превышения указанной длины участка работ, ВЛ должна быть разделена на электрически не связанные между собой участки с установкам на каждом из них базового заземления.

При выполнении таких работ на участке одиночного следования линии, а также всех видов работ в зоне слабого действия наведенного напряжения длина участка производства работ, не ограничивается.

Совмещение работ в зоне сильного действия наведенного напряжения на участках совместимого и одиночного следования допускается только при разделении линии на электрически несвязанные участки.

6.1.64. При прохождении ВЛ на участке одиночного следования на территории разных предприятий на каждом участке работ должен устанавливаться отдельный базовый заземлитель.

6.1.65. До начала работ под наведенным напряжением должны быть выровнены потенциалы провода, опор и их элементов, монтажных канатов, машин и механизмов путем заземления их на общий заземлитель. В этом случае провод следует заземлять в последнюю очередь — после сборки такелажной схемы на уровне земли и ее заземления. Разбирать такелажную схему необходимо в обратной последовательности.

6.1.66. Работы, связанные с прикосновением к опущенному до земли проводу, должны проводиться с использованием электрозащитных средств или с металлической площадки, соединенной с проводом для выравнивания потенциалов. Запрещается входить на площадку или сходить с нее, а также подавать металлические предметы, стоя на земле без диэлектрической обуви.

6.1.67. Перед разрезанием провода, его необходимо заземлить с обеих сторон от места разрыва на контур заземления опоры или, при выполнении таких работ в пролете — на общий групповой заземлитель, на который должны заземлены также монтажные канаты, машины и механизмы.

6.1.68. Для защиты от напряжения шага после заземления провода на месте работ при приближении к заземлителю на расстояние менее 3 м необходимо применять диэлектрическую обувь.

6.1.69. При монтаже и замене проводов находящихся под наведенным напряжением, все работы, связанные с прикосновением к проводу, машинам и механизмам, должны выполняться с заземлением их на месте работ и применением электрозащитных средств. Перед раскаткой заземлять провод непосредственно у барабана не требуется.

6.1.70. Подъем и опускание провода должны проводиться с заземлением его на каждой опоре, где производится работа, так, чтобы длина участка не превышала 2 км. Натяжку и визирование провода следует выполнять с заземлением его на анкерной опоре, через которую производится натяжка.

6.1.71. Перекладка провода из раскаточных роликов в зажимы должна выполняться после заземления его на месте работ или на соседней опоре. Для провода, лежащего на металлических роликах или в поддерживающих зажимах, достаточно заземлить их на контур заземления опоры, а при наличии естественного контакта между ними установка дополнительного заземления на месте работ не требуется. Смежный анкерный пролет, в котором перекладка провода уже закончена, следует считать находящимся под напряжением.

6.1.72. До работ по соединению проводов в петлях анкерных опор ВЛ 110 кВ и выше их следует закреплять за провода или за натяжные изолирующие подвески (но не ближе, чем за четвертый изолятор от траверсы), а на ВЛ 35 кВ и ниже — только за провода.

6.1.73. К соединению проводов в пролетах анкерных опор можно приступать только после полного окончания работ в смежных анкерных пролетах и снятия всех заземлений. Линия должна быть заземлена в одном месте — на анкерной опоре, где осуществляются работы, с заземлением концов соединяемых проводов на контур заземления опоры. До установки заземлений следует соблюдать повышенную осторожность и не допускать приближения к незаземленным проводам из-за наличия на них наведенного электростатического потенциала.

6.1.74. Работы на оборудовании электростанций и подстанций, находящемся под наведенным напряжением, необходимо выполнять с установкой на спуски проводов со стороны ВЛ по одному переносному заземлению или с включением заземляющих ножей на обходном разъединителе, если на нем не производятся работы. Установку и снятие переносных заземлений необходимо выполнять при условии включенных заземляющих ножей в сторону линии.

6.1.75. При работах в электроустановках под наведенным напряжением с применением телескопических вышек и гидроподъемников рабочая площадка должна быть соединена с заземленным на месте работ проводом — перемычкой из гибкого медного провода при помощи специальной штанги, а сам механизм заземлен на общий с проводом заземлитель. Сечение перемычки и заземляющего проводника должно быть не менее 25 мм2. Механизмы должны быть оснащены инвентарными заземлителями, на рабочих площадках должны быть обозначены места для присоединения перемычек, тщательно очищенных от краски, ржавчины и загрязнений.

Наведенное напряжение и меры защиты

Правилами техники безопасности (ПТБ) при эксплуатации электроустановок определены меры безопасности во время работ на воздушных линиях (ВЛ) электропередачи , на которых наводится дополнительное напряжение от соседних работающих линий. Отдельно выделены меры безопасности при работах на таких ВЛ, когда заземление их в соответствии с общими требованиями правил не позволяет снизить уровень наводящегося на отключённых проводах потенциала ниже 25 В.

Однако продолжают иметь место случаи поражения обслуживающего персонала электрическим током наведенного напряжения, которые являются результатом недопонимания природы возникновения и механизма проявления этого напряжения. Особенность его проявления состоит в сохранении опасности поражения электрическим током при прикосновении даже к заземлённому по правилам проводу.

Известно, что на любой ВЛ, проходящей параллельно с другими ВЛ , непрерывно наводится сторонний потенциал , обусловленный взаимным влиянием электромагнитных полей этих линий друг на друга. Значение потенциала зависит от рабочего напряжения, токов нагрузки, расстояния между фазными проводами линий и длины участка параллельного их расположения.

Наведенный на каждой из таких линий потенциал (наведенное напряжение) можно условно представить в виде суммы двух составляющих: электростатической и электромагнитной.

Электростатическая составляющая наведенного напряжения на проводах отключённой ВЛ обусловлена воздействием на них электрического поля остающейся в работе соседней (влияющей) линии и при сохранении определённых ПУЭ конструктивных параметров участка параллельного следования зависит только от уровня напряжения влияющей линии. Значение этой составляющей одинаково по всей длине отключённой ВЛ (рис. 1) и определяется по формуле:

Uэ = k Uр.в.

где  k   –  коэффициент ёмкостной связи линий ;

Uр.в. – рабочее напряжение влияющей линии.

Рис. 1. Диаграмма распределения электростатической  составляющей наведенного напряжения.

 

 

Электростатическая составляющая наведенного напряжения снижается до безопасного уровня по всей длине линии при заземлении её в любой, хотя бы одной точке. Следовательно, воздействие этой составляющей полностью устраняется при заземлении отключенной ВЛ по концам (на подстанциях) и на месте производства работ согласно ПТБ.

Совсем иначе проявляет себя электромагнитная составляющая наведенного напряжения , возникновение которой обусловлено суммарным влиянием магнитных полей , создаваемых токами фазных проводов влияющей линии.

Наводимая на отключенной линии ЭДС определяется выражением:

E=MLI

где M  – коэффициент индуктивной связи фазных проводов линии при частоте 50 Гц ;

L   – длина участка параллельного следования линии ;

I  – ток нагрузки влияющей линии.

Коэффициент индуктивной связи для каждого конкретного «коридора» линий практически не меняется. В связи с этим значение наведенной ЭДС обусловлено только длинной участка параллельного следования линий и током нагрузки влияющей линии и не зависит от уровня рабочих напряжений каждой из ВЛ.

При этом потенциал (напряжение относительно земли) любой точки , например x , определяется выражением:

U=- E/L *x + E/2

где E  – наведенная на проводе ЭДС;

x – расстояние от начала линии до точки x.

Отсюда следует, что в начале линии (при x=0 ) электромагнитная составляющая наведенного напряжения Uн=+E/2, в конце линии Uк=-E/2 (при x= L) в середине линии Uср=0 (при x=L/2).

Особенностью проявления электромагнитной составляющей наведенного напряжения  является неизменность её значения независимо от того, изолирован провод от земли или заземлён в одном или даже в нескольких местах.

При изменении числа точек заземления на ВЛ меняется лишь положение точки нулевого потенциала на ней. Специфичность именно этого проявления наведенного напряжения и обусловлены требования ПТБ.

Рис. 2. Диаграмма распределения электромагнитной составляющей наведенного напряжения на отключённой ВЛ в зависимости от места установки на ней защитных заземлений.

На рис. 2 приведены характерные примеры распределения электромагнитной составляющей наведенного напряжения (потенциала) на отключённой ВЛ в зависимости от места установки защитных заземлений. Как видно из диаграмм , при однократном заземлении ВЛ точка нулевого потенциала совпадает с точкой заземления.

С учётом изложенного представлено графическое обоснование опасности организации одновременно двух и более рабочих мест на ВЛ , находящейся в зоне наведения электромагнитной составляющей напряжения. Например , бригада работает в точке С , линия согласно правилам заземлена только в одной этой точке , где напряжение равно нулю (рис.3а).

Если теперь для подготовки второго рабочего места установить защитное заземление в другой точке D , то нулевой потенциал переместится на участок между двумя этими заземлениями (рис. 3б). При этом напряжение в местах заземления (точки С и D) может превысить допустимый уровень , и работающие там люди подвергнутся опасности поражения электрическим током.

Аналогичный эффект  проявляется и при производстве работ на линейном разъединителе , находящемся под наведенным напряжением ВЛ. Заземление разъединителя со стороны линии в этом случае гарантирует электробезопасность только при условии , что эта линия нигде больше не заземлена (см. рис. 2б, д).


Если установить дополнительное заземление на каком-либо другом участке , например , включить заземляющие ножи на подстанции с другого конца линии , то уровень наведенного напряжения на линейном разъединителе в месте производства работ «подскочит» до максимума (см. рис. 2г).

 

Рис. 3. Примеры распределения электромагнитной составляющей напряжения на отключённой ВЛ при работе ремонтной бригады в различных условиях.

Проявления наведенного напряжения вынуждают эксплутационный персонал резко сокращать фронт работы на ВЛ (до одной бригады) , находящихся в зоне усиленного действия этого напряжения. Разделение линии на отдельные электрически несвязанные участки путём разрезания шлейфов также вызывает дополнительные затраты времени на поочерёдное разрезание и последующее их восстановление. Однако необходимость обеспечения безопасности линейного персонала обязывает считаться с этими фактами.

Вместе с тем одной из альтернативных мер, снимающих практически все ограничения в расширении фронта производства работ во всех случаях (при сохранении безопасности линейного персонала) , является выполнение работ под напряжением.

При подготовке рабочего места на ВЛ следует обращать особое внимание на надёжность контактов защитных заземлений с фазными проводами и заземлителем. Нельзя забывать, что при случайной потере контакта (разземлении линии) точка нулевого потенциала в то же мгновение может изменить своё местоположение , а напряжение на рабочем месте превысить допустимое значение  Uс  (рис 4). Следовательно, для гарантии безопасности в месте производства работ целесообразно устанавливать параллельно два заземления.

Рис. 4. Диаграмма распределения электромагнитной составляющей наведенного напряжения при заземлении линии в точке С и при её разземлении

 

Итак, наибольшего значения электромагнитная составляющая наведенного напряжения достигает на границах участка взаимного влияния линий (в общем случае – на отключённых линейных разъединителях). Именно в этих точках, непосредственно на спуске шины заземления линейного разъединителя или на первой от подстанции опоре, следует производить измерения при включённых с обеих концов линии заземляющих ножах.

Класс напряжения используемых для этого вольтметров необходимо подбирать по ожидаемому уровню наведенного напряжения. В первом приближении можно использовать вольтметр с пределом измерения до 0,5í1,0 кВ.

Пересчёт результатов измерения на условия максимальных нагрузок влияющей линии можно провести по формуле , полученной из соотношения:

где   Uизм – измеренное наведенное напряжение ;

Iизм   – ток нагрузки влияющей ВЛ в момент измерения ;

Iмакс  – максимальный допустимый ток нагрузки влияющей линии.

Следует отметить, что включенные заземляющие ножи, рама разъединителя, соединительные провода и вольтметр во время измерений могут находиться под опасным напряжением. В целях обеспечения безопасности персонала, производящего измерения, соединять схему измерения с фазными проводами линии следует только после сборки схемы измерения. При необходимости переключения пределов шкалы или замены вольтметра предварительно необходимо отсоединить схему измерения от провода ВЛ.

Персонал должен пользоваться диэлектрическими ботами и перчатками. Используемые при измерениях провода должны иметь изоляцию, рассчитанную на напряжение 1 кВ.

 

Специалисты Динского РЭС получили статус инструкторов для работы под напряжением

Специалисты Динского РЭС получили статус инструкторов для работы под напряжением

Специалисты Динского РЭС Краснодарского филиала компании «Россети Кубань» в рамках пилотного образовательного проекта получили статус инструкторовдля работы под напряжением до 1000 вольт. Курсы практической подготовки прошли в Центре работы под напряжением в городе Заинск Республики Татарстан.

Статус инструкторов дает правоэнергетикам обучать передовым практикам кубанских коллег.

В ходе обучения энергетики освоили использование виртуальных и интерактивных технологий в учебном процессе. Этот опыт им предстоит передавать в рамках подготовки персонала к работе под напряжением на базе корпоративного энергетического института в Краснодаре.

– Работы на воздушных линиях электропередачи под наведенным напряжением являются одними  из  самых опасных, – рассказал участник пилотного образовательного проекта, мастер Динского РЭС Сергей Бояркин. – Практико-прикладной характер образовательной программы позволил нам максимально эффективно освоить теоретические, технические, организационные и даже психологические основы проведения работ под наведенным напряжением.

Наведенное напряжение – это напряжение более 25 вольт, возникающее в отключенных линиях электропередачи, которое проходит по всей длине или на отдельных участках вблизи действующих ЛЭП. Передовые технологии и практики работы под напряжением позволяют выполнять большой объем технических мероприятий без отключения электроэнергии у потребителей.

ПАО «Россети Кубань» отвечает за транспорт электроэнергии по сетям 110 кВ и ниже на территории Краснодарского края и Республики Адыгея. Входит в группу «Россети». В составе энергосистемы 11 электросетевых филиалов (Краснодарские, Сочинские, Армавирские, Адыгейские, Тимашевские, Тихорецкие, Ленинградские, Славянские, Юго-Западные, Лабинские, Усть-Лабинские). Общая протяженность линий электропередачи достигает 90 тыс. км. Площадь обслуживаемой территории – 83,8 тыс. кв. км с населением более 6 млн человек. «Россети Кубань» – крупнейший налогоплательщик  региона. Телефон горячей линии: 8-800-100-15-52 (звонок по России бесплатный).

Компания «Россети» является оператором одного из крупнейших электросетевых комплексов в мире. Управляет 2,35 млн км линий электропередачи, 507 тыс. подстанций трансформаторной мощностью более 792 ГВА. В 2019 году полезный отпуск электроэнергии потребителям составил 763 млрд кВт·ч. Численность персонала группы компаний «Россети» – 220 тыс. человек. Имущественный комплекс ПАО «Россети» включает 35 дочерних и зависимых обществ, в том числе 15 межрегиональных, и магистральную сетевую компанию. Контролирующим акционером является государство в лице Федерального агентства по управлению государственным имуществом РФ, владеющее 88,04 % долей в уставном капитале.

Контакты:

Дирекция по связям с общественностью ПАО «Россети Кубань»

Тел.: (861) 212-24-68; e-mail: [email protected]

 

Голос опыта: понимание индуцированного напряжения

Электроэнергетикам потребовалось много лет, чтобы разобраться в индуцированном напряжении. Когда я начал работать в 1960-х, мне объяснили, что напряжение, остающееся на обесточенных линиях, было статическим, которое нужно было сбросить, иначе оно могло быть смертельным. Теперь, когда я говорю с группами о временном системном заземлении для защиты сотрудников, я иногда все еще слышу термин «статическое напряжение», используемый для описания того, что на самом деле является наведенным напряжением от соседней линии, находящейся под напряжением.Даже сегодня не все в отрасли полностью понимают наведенное напряжение.

Итак, что такое наведенное напряжение? Вот некоторые вещи, которые должны понимать специалисты по безопасности и эксплуатации. Электромагнитное поле вокруг проводника под напряжением создает емкостную и магнитную связь со всеми близлежащими объектами в пределах электромагнитного поля. Уровень напряжения проводника под напряжением и физическая длина обесточенного проводника, который подвергается воздействию проводника (источника) под напряжением, будут определять величину напряжения на обесточенном проводнике или оборудовании.Обесточенный проводник или часть оборудования будут оставаться под напряжением, пока источник остается под напряжением, а обесточенное оборудование остается незаземленным. Правильно установленные временные системные площадки безопасности можно использовать для создания уравновешенной рабочей зоны для сотрудников.

Наведенное напряжение на обесточенном оборудовании не статично, и его нельзя сбросить. Установленные защитные заземления системы просто обеспечивают проводящее соединение индуцированного напряжения с землей. После удаления заземления индуцированное напряжение мгновенно возвращается к точно такой же величине напряжения.Это напряжение 60 циклов в секунду в установившемся состоянии, потому что нет другого пути, по которому может течь электричество, кроме изолированного проводника или оборудования под напряжением. Если заземление применяется к обесточенным проводникам, напряжение немедленно упадет почти до нуля, но теперь физика изменилась, и в заземлении системы устанавливается ток. Сила тока, протекающего в заземляющих устройствах, определяется величиной наведенного напряжения на обесточенном оборудовании до установки заземления, а также сопротивлением заземляющего устройства и земли.Кроме того, чем больше наборов заземлений применяется к обесточенной линии, тем меньше ток протекает в каждом наборе заземлений.

Существенные изменения
За последние 10 лет произошло множество травм и смертельных случаев, связанных с неспособностью контролировать наведенное напряжение. В 2014 году в правила OSHA 29 CFR 1910.269 была внесена пара значительных изменений в попытке решить проблемы наведенного напряжения.

Во-первых, давайте взглянем на пункт 1910.269 (m), «Выключение линий и оборудования для защиты сотрудников.«Правило всегда гласило, что работодатель должен обеспечить установку заземления системы. В частности, параграф 1910.269 (m) (3) (vii) гласит следующее: «Работодатель должен обеспечить установку защитных оснований в соответствии с требованиями параграфа (n) этого раздела».

До тех пор, пока не будут заземлены обесточенные линии и оборудование, параграф 1910.269 (n) требует, чтобы сотрудники придерживались минимального подхода и считали, что обесточенные линии и оборудование должны быть под напряжением. Согласно 1910.269 (n) (3), должна быть установлена ​​эквипотенциальная зона.В абзаце указано следующее: «Эквипотенциальная зона. В таких местах должны быть размещены временные защитные площадки и организованы таким образом, чтобы работодатель мог продемонстрировать, что они не допустят воздействия на каждого работника опасной разницы в электрическом потенциале ».

В попытке управлять опасной энергией и наведенным напряжением, значительное изменение в 1910.269 (q), «Воздушные линии и работа без оборудования под напряжением», осталось практически незамеченным, когда новое правило 1910.269 было опубликовано в 2014 году, и никакого внимания не было. к нему во время первых вебинаров о новом правиле.Объяснение изменения можно найти в 1910.269 (q) (2) (iv). До обновления 2014 года, если бригады работали или устанавливали проводники параллельно линиям под напряжением, заземления системы требовались на расстоянии не менее 2 миль друг от друга. Таким образом, при работе на заземленных линиях сотрудники никогда не будут находиться более чем в миле от набора временных защитных сооружений. Как выясняется, 1 миля от ряда защитных сооружений системы на полосе отвода 345 кВ или 500 кВ может быть слишком далеко, что может подвергнуть сотрудников опасной разнице потенциалов, если они коснутся обесточенных линий или оборудование.

Обновленный 1910.269 (q) (2) (iv) теперь гласит следующее: «Перед тем, как сотрудники установят линии, параллельные существующим линиям, находящимся под напряжением, работодатель должен определить приблизительное напряжение, которое будет индуцировано в новых линиях, или работа должна исходить из предположения, что индуцированное напряжение опасно. Если работодатель не может продемонстрировать, что линии, которые устанавливают работники, не подвержены наведению опасного напряжения, или если линии не рассматриваются как находящиеся под напряжением, в таких местах должны быть размещены временные защитные заземления и организованы таким образом, чтобы работодатель мог демонстрация предотвратит воздействие на каждого сотрудника опасной разницы в электрическом потенциале.”

Примечание 1 к параграфу 1910.269 (q) (2) (iv) гласит: «Если работодатель не принимает мер предосторожности для защиты сотрудников от опасностей, связанных с непроизвольной реакцией от поражения электрическим током, существует опасность, если индуцированное напряжение достаточно, чтобы пропустить ток. 1 миллиампер через резистор на 500 Ом. Если работодатель защищает сотрудников от травм из-за непроизвольной реакции на поражение электрическим током, существует опасность, если результирующий ток будет более 6 миллиампер ».

Вы могли заметить, что текст 1910 г.269 ​​(n) (3) был скопирован и добавлен к 1910.269 (q) (2) (iv) в попытке обеспечить защиту сотрудников от опасных перепадов потенциала. Методы определения местоположения заземления на проводниках могут потребовать заземления чаще, чем на 2 мили, чтобы уменьшить риски разницы потенциалов. После установки проводов дополнительные защитные заземления системы снизят индуцированное напряжение и будут соответствовать нормативам.

После разговоров со многими рабочими об индуцированном напряжении возникло мнение, что после установки заземления вся линия обесточивается.Наука говорит нам, что защитное заземление системы — единственное место на заземленной линии, где напряжение относительно земли равно нулю. В случае наведенного напряжения, чем дальше вы находитесь от временного заземления, тем больше вероятность разницы потенциалов между заземленными проводниками и другими поверхностями — отсюда и изменение правил. Обратите внимание, что когда сотрудники работают в заземленной корзине крана или JLG в заземленной цепи на полосе отвода или на подстанции, в промежутке между автобусом и платформой будет возникать разность потенциалов.Эти токопроводящие платформы должны быть соединены с заземленными проводниками, чтобы закрыть этот разрыв и защитить рабочих в корзине от разницы потенциалов.

Кроме того, даже когда оборудование заземлено, а шина или проводники заземлены, могут существовать циркулирующие токи заземления, связанные с наведенным напряжением и путем к земле. Заземляющее оборудование в другом месте, даже на большой подстанции, может создать опасные условия на территории.

Заключение
Мы должны помнить, что электричество не идет только по пути наименьшего сопротивления, как мне говорили много лет назад.Вместо этого электричество пойдет по всем проводящим путям. Закон Кирхгофа о делении тока в параллельных цепях помогает нам понять, что величина тока, протекающего по пути, определяется импедансом и сопротивлением пути. Требуется всего около 50 вольт переменного тока, чтобы проникнуть через кожу человека, и от 30 до 50 миллиампер, чтобы стать фатальным для человека. У людей в электрической цепи всего лишь резистор сопротивлением 1000 Ом. Все сотрудники должны быть знакомы с законом параллельных сопротивлений и законом Ома.

Об авторе: Дэнни Рейнс, CUSP, консультант по безопасности, распределению и передаче, ушел на пенсию из Georgia Power после 40 лет службы и открыл Raines Utility Safety Solutions LLC, обеспечивающий обучение соблюдению, оценку рисков и программы наблюдения за безопасностью. Он также является аффилированным инструктором в Техническом исследовательском центре Джорджии OSHA Outreach в Атланте.

Закон индукции Фарадея: Закон Ленца

Задачи обучения

К концу этого раздела вы сможете:

  • Рассчитайте ЭДС, ток и магнитные поля, используя закон Фарадея.
  • Объясните физические результаты закона Ленца.

Закон Фарадея и Ленца

Эксперименты Фарадея показали, что ЭДС, вызванная изменением магнитного потока, зависит только от нескольких факторов. Во-первых, ЭДС прямо пропорциональна изменению магнитного потока Δ Φ . Во-вторых, ЭДС является наибольшей, когда изменение во времени Δ t наименьшее, то есть ЭДС обратно пропорциональна Δ t . Наконец, если катушка имеет Н и витков, будет создаваться ЭДС, которая в Н в раз больше, чем для одиночной катушки, так что ЭДС прямо пропорциональна Н .Уравнение для ЭДС, вызванной изменением магнитного потока, равно

[латекс] \ text {emf} = — N \ frac {\ Delta \ Phi} {\ Delta t} \\ [/ latex].

Это соотношение известно как закон индукции Фарадея . Обычно единицами измерения ЭДС являются вольты. Знак минус в законе индукции Фарадея очень важен. Минус означает, что ЭДС создает ток I и магнитное поле B, которые препятствуют изменению магнитного потока Δ Φ — это известно как закон Ленца . Направление (обозначенное знаком минус) ЭДС настолько важно, что оно было названо законом Ленца в честь русского Генриха Ленца (1804–1865), который, подобно Фарадею и Генри, независимо исследовал аспекты индукции.Фарадей знал о направлении, но Ленц так ясно изложил его, что ему приписывают его открытие. (См. Рисунок 1.)

Рис. 1. (a) Когда стержневой магнит вставляется в катушку, сила магнитного поля в катушке увеличивается. Ток, наведенный в катушке, создает другое поле в направлении, противоположном направлению стержневого магнита, чтобы противодействовать увеличению. Это один из аспектов закона Ленца: индукция препятствует любому изменению потока. (b) и (c) — две другие ситуации. Убедитесь сами, что направление показанной наведенной катушки B действительно противостоит изменению магнитного потока и что показанное направление тока согласуется с RHR-2.

Стратегия решения проблем закона Ленца

Чтобы использовать закон Ленца для определения направлений индуцированных магнитных полей, токов и ЭДС:

  1. Сделайте набросок ситуации для использования при визуализации и записи направлений.
  2. Определите направление магнитного поля Б.
  3. Определите, увеличивается или уменьшается поток.
  4. Теперь определите направление индуцированного магнитного поля B. Оно противодействует изменению магнитного потока путем добавления или вычитания из исходного поля.
  5. Используйте RHR-2, чтобы определить направление индуцированного тока I, ответственного за индуцированное магнитное поле B.
  6. Направление (или полярность) наведенной ЭДС теперь будет управлять током в этом направлении и может быть представлено как ток, выходящий из положительного вывода ЭДС и возвращающийся к ее отрицательному выводу.

Для практики примените эти шаги к ситуациям, показанным на Рисунке 1, и другим, которые являются частью следующего текстового материала.

Применение электромагнитной индукции

Существует множество применений закона индукции Фарадея, которые мы исследуем в этой и других главах. В этот момент позвольте нам упомянуть несколько, которые имеют отношение к хранению данных и магнитным полям. Очень важное приложение связано с записью аудио и видео на магнитные ленты . Пластиковая лента, покрытая оксидом железа, проходит мимо записывающей головки. Эта записывающая головка представляет собой круглое железное кольцо, на которое намотана катушка с проволокой — электромагнит (рис. 2).Сигнал в виде переменного входного тока от микрофона или камеры поступает на записывающую головку. Эти сигналы (которые являются функцией амплитуды и частоты сигнала) создают переменные магнитные поля на записывающей головке. Когда лента движется мимо записывающей головки, ориентация магнитного поля молекул оксида железа на ленте изменяется, таким образом записывая сигнал. В режиме воспроизведения намагниченная лента проходит мимо другой головки, аналогичной по конструкции записывающей головке. Различная ориентация магнитного поля молекул оксида железа на ленте индуцирует ЭДС в проволочной катушке в воспроизводящей головке.Затем этот сигнал отправляется на громкоговоритель или видеоплеер.

Рис. 2. Головки записи и воспроизведения, используемые с аудио- и видеомагнитными лентами. (кредит: Стив Джурветсон)

Аналогичные принципы применимы и к жестким дискам компьютеров, но с гораздо большей скоростью. Здесь записи находятся на вращающемся диске с покрытием. Исторически считывающие головки создавались по принципу индукции. Однако входная информация передается в цифровой, а не аналоговой форме — на вращающемся жестком диске записывается серия нулей или единиц.Сегодня большинство считывающих устройств с жестких дисков не работают по принципу индукции, а используют технологию, известную как гигантское магнитосопротивление . (Открытие того факта, что слабые изменения магнитного поля в тонкой пленке из железа и хрома могут вызывать гораздо большие изменения электрического сопротивления, было одним из первых крупных успехов нанотехнологии.) Еще одно применение индукции можно найти на магнитной полосе на магнитной полосе. на оборотной стороне вашей личной кредитной карты, которая использовалась в продуктовом магазине или в банкомате.Это работает по тому же принципу, что и аудио- или видеолента, упомянутая в последнем абзаце, в которой голова считывает личную информацию с вашей карты.

Другое применение электромагнитной индукции — это когда электрические сигналы должны передаваться через барьер. Рассмотрим кохлеарный имплант , показанный ниже. Звук улавливается микрофоном на внешней стороне черепа и используется для создания переменного магнитного поля. Ток индуцируется в приемнике, закрепленном в кости под кожей, и передается на электроды во внутреннем ухе.Электромагнитная индукция может использоваться и в других случаях, когда электрические сигналы должны передаваться через различные среды.

Рис. 3. Электромагнитная индукция, используемая при передаче электрического тока через среды. Устройство на голове ребенка индуцирует электрический ток в приемнике, закрепленном в кости под кожей. (кредит: Бьорн Кнетч)

Еще одна современная область исследований, в которой электромагнитная индукция успешно реализуется (и имеет значительный потенциал), — это транскраниальное магнитное моделирование.Множество расстройств, включая депрессию и галлюцинации, можно объяснить нерегулярной локальной электрической активностью в головном мозге. В транскраниальной магнитной стимуляции быстро меняющееся и очень локализованное магнитное поле помещается рядом с определенными участками, идентифицированными в головном мозге. В идентифицированных участках индуцируются слабые электрические токи, которые могут привести к восстановлению электрических функций в тканях мозга.

Апноэ сна («остановка дыхания») поражает как взрослых, так и младенцев (особенно недоношенных детей и может быть причиной внезапной детской смерти [SID]).У таких людей дыхание может многократно останавливаться во время сна. Прекращение действия более чем на 20 секунд может быть очень опасным. Инсульт, сердечная недостаточность и усталость — вот лишь некоторые из возможных последствий для человека, страдающего апноэ во сне. У младенцев проблема заключается в задержке дыхания на это более длительное время. В одном из типов мониторов, предупреждающих родителей о том, что ребенок не дышит, используется электромагнитная индукция. Проволока, намотанная вокруг груди младенца, пропускает через нее переменный ток. Расширение и сжатие грудной клетки младенца во время дыхания изменяет площадь спирали.В расположенной рядом катушке датчика индуцируется переменный ток из-за изменяющегося магнитного поля исходного провода. Если ребенок перестанет дышать, наведенный ток изменится, и родители могут быть предупреждены.

Подключение: сохранение энергии

Закон Ленца является проявлением сохранения энергии. Индуцированная ЭДС создает ток, который противодействует изменению потока, потому что изменение потока означает изменение энергии.Энергия может входить или уходить, но не мгновенно. Закон Ленца — следствие. Когда изменение начинается, закон гласит, что индукция противодействует и, таким образом, замедляет изменение. Фактически, если бы индуцированная ЭДС была в том же направлении, что и изменение потока, была бы положительная обратная связь, которая не давала бы нам бесплатную энергию из любого видимого источника — закон сохранения энергии был бы нарушен.

Пример 1. Расчет ЭДС: насколько велика наведенная ЭДС?

Рассчитайте величину наведенной ЭДС, когда магнит, изображенный на Рисунке 1 (а), вдавливается в катушку, учитывая следующую информацию: одноконтурная катушка имеет радиус 6.00 см, а среднее значение B cos θ (это дано, поскольку поле стержневого магнита сложное) увеличивается с 0,0500 Тл до 0,250 Тл за 0,100 с.

Стратегия

Чтобы найти величину ЭДС , мы используем закон индукции Фарадея, как указано в [latex] \ text {emf} = — N \ frac {\ Delta \ Phi} {\ Delta t} \\ [/ latex], но без знака минус, указывающего направление:

[латекс] \ text {emf} = N \ frac {\ Delta \ Phi} {\ Delta t} \\ [/ latex].

Решение

Нам дано, что N = 1 и Δ t = 0.100 с, но мы должны определить изменение потока Δ Φ , прежде чем мы сможем найти ЭДС. Поскольку площадь петли фиксирована, мы видим, что

ΔΦ = Δ ( BA cos θ ) = AΔ ( B cos θ ).

Теперь Δ ( B cos θ ) = 0,200 Тл, поскольку было задано, что B cos θ изменяется от 0,0500 до 0,250 Тл. Площадь контура A = πr2 = (3,14…) ( 0,060 м) 2 = 1,13 × 10 −2 м 2 .{2} \ right) \ left (0.200 \ text {T} \ right)} {0.100 \ text {s}} = 22.6 \ text {mV} \\ [/ latex].

Обсуждение

Хотя это напряжение легко измерить, его явно недостаточно для большинства практических приложений. Больше петель в катушке, более сильный магнит и более быстрое движение делают индукцию практическим источником напряжения.

Исследования PhET: Электромагнитная лаборатория Фарадея

Поиграйте с стержневым магнитом и катушками, чтобы узнать о законе Фарадея.Поднесите стержневой магнит к одной или двум катушкам, чтобы лампочка загорелась. Просмотрите силовые линии магнитного поля. Измеритель показывает направление и величину тока. Просмотрите линии магнитного поля или используйте измеритель, чтобы показать направление и величину тока. Вы также можете играть с электромагнитами, генераторами и трансформаторами!

Щелкните, чтобы загрузить симуляцию. Запускать с использованием Java.

Сводка раздела

Концептуальные вопросы

  1. Человек, работающий с большими магнитами, иногда помещает голову в сильное поле.Она сообщает, что у нее кружится голова, когда она быстро поворачивает голову. Как это может быть связано с индукцией?
  2. Ускоритель частиц отправляет заряженные частицы с высокой скоростью по откачанной трубе. Объясните, как катушка с проволокой, намотанная вокруг трубы, может обнаруживать прохождение отдельных частиц. Нарисуйте график выходного напряжения катушки при прохождении через нее одиночной частицы.

Задачи и упражнения

1. Как показано на Рисунке 5 (а), каково направление тока, индуцируемого в катушке 2: (а) Если ток в катушке 1 увеличивается? (б) Если ток в катушке 1 уменьшается? (c) Если ток в катушке 1 постоянный? Ясно покажите, как вы следуете шагам из приведенной выше стратегии решения проблем для закона Ленца .

Рис. 5. (a) Катушки лежат в одной плоскости. (б) Проволока находится в плоскости катушки.

2. Как показано на Рисунке 5 (b), в каком направлении индуцируется ток в катушке: (a) Если ток в проводе увеличивается? (б) Если ток в проводе уменьшится? (c) Если ток в проводе внезапно меняет направление? Ясно покажите, как вы следуете шагам из приведенной выше стратегии решения проблем для закона Ленца .

3. Как показано на рисунке 6, каковы направления токов в катушках 1, 2 и 3 (предположим, что катушки лежат в плоскости цепи): (a) Когда переключатель в первый раз замыкается? (б) Когда переключатель был замкнут в течение длительного времени? (c) Сразу после размыкания переключателя?

Рисунок 6.

4. Повторите предыдущую проблему с перевернутой батареей.

5. Убедитесь, что единицами измерения Δ Φ / Δ т являются вольты. То есть показать, что 1 Тл м 2 / с = 1 В.

6. Предположим, что 50-витковая катушка лежит в плоскости страницы в однородном магнитном поле, направленном внутрь страницы. Змеевик изначально имел площадь 0,250 м 2 . Он растягивается, чтобы не было площади за 0,100 с. Каковы направление и величина наведенной ЭДС, если однородное магнитное поле имеет напряженность 1?50 т?

7. (a) Техник МРТ перемещает свою руку из области очень низкой напряженности магнитного поля в поле 2,00 Тл сканера МРТ, указывая пальцами в направлении поля. Найдите среднюю ЭДС, индуцированную в его обручальном кольце, учитывая его диаметр 2,20 см и предполагая, что для его перемещения в поле требуется 0,250 с. (б) Обсудите, может ли этот ток существенно изменить температуру кольца.

8. Integrated Concepts Ссылаясь на ситуацию в предыдущей задаче: (a) Какой ток индуцируется в кольце, если его сопротивление равно 0.0100 Ом? (б) Какая средняя мощность рассеивается? (c) Какое магнитное поле индуцируется в центре кольца? (d) Каково направление индуцированного магнитного поля относительно поля МРТ?

9. ЭДС индуцируется вращением катушки с 1000 витками диаметром 20,0 см в магнитном поле Земли 5,00 × 10 −5 Тл. Какая средняя ЭДС индуцируется, учитывая, что плоскость катушки изначально перпендикулярна полю Земли и повернута параллельно полю за 10,0 мс?

10.Катушка с 500 витками радиусом 0,250 м поворачивается на одну четверть оборота за 4,17 мс, первоначально ее плоскость перпендикулярна однородному магнитному полю. (Это 60 об / с.) Найдите напряженность магнитного поля, необходимую для индукции средней ЭДС 10 000 В.

11. Integrated Concepts Примерно как ЭДС, наведенная в петле на рисунке 5 (b), зависит от расстояния центра петли от провода?

12. Integrated Concepts (a) Молния создает быстро меняющееся магнитное поле.Если болт ударяется о землю вертикально и действует как ток в длинном прямом проводе, он вызывает напряжение в петле, выровненной, как показано на рисунке 5 (b). Какое напряжение индуцируется в петле диаметром 1,00 м и 50,0 м от удара молнии 2,00 × 10 6 , если ток падает до нуля за 25,0 мкс? (б) Обсудите обстоятельства, при которых такое напряжение может привести к заметным последствиям.

Глоссарий

Закон индукции Фарадея:
средство вычисления ЭДС в катушке из-за изменения магнитного потока, заданное как [latex] \ text {emf} = — N \ frac {\ Delta \ Phi} {\ Delta t} \\ [/ latex]
Закон Ленца:
знак минус в законе Фарадея, означающий, что ЭДС, индуцированная в катушке, противодействует изменению магнитного потока.

Избранные решения проблем и упражнения

1.(a) CCW (b) CW (c) Отсутствие индуцированного тока

3. (a) 1 против часовой стрелки, 2 против часовой стрелки, 3 по часовой стрелке (b) 1, 2 и 3 без тока индуцированного (c) 1 CW, 2 CW, 3 CCW

7. (a) 3,04 мВ (b) В качестве нижнего предела для кольца, оценка R = 1,00 мОм. Передаваемое тепло составит 2,31 мДж. Это небольшое количество тепла.

9. 0,157 В

11. пропорционально [латексу] \ frac {1} {r} \\ [/ latex]

23.2: Индукция в движущемся проводнике

Если мы определим проволочную петлю, существует два способа изменения магнитного потока, проходящего через эту петлю:

  1. Магнитное поле может изменять величину или направление, как мы видели в примере 23 .1.1 .
  2. Петля может менять размер или ориентацию относительно магнитного поля.

В этом разделе мы исследуем последний случай, иногда называемый «двигательной ЭДС», поскольку индуцированное напряжение является результатом движения контура, в котором индуцируется напряжение.

Движение штанги по двум параллельным рельсам

Рассмотрим U-образную направляющую в однородном магнитном поле, по которой стержень может скользить без трения, как показано на рисунке \ (\ PageIndex {1} \).Полоса длины \ (L \) движется вправо с постоянной скоростью \ (v \).

Рисунок \ (\ PageIndex {1} \): U-образная направляющая, по которой может скользить полоса длиной \ (L \). Система погружена в магнитное поле, указывающее за пределы страницы. Полоса движется вправо с постоянной скоростью \ (v \).

Штанга и рельсы образуют замкнутый контур площадью:

\ [\ begin {выровнено} A (t) = Lw (t) = Lvt \ end {выровнено} \]

, который со временем увеличивается. Величина потока через контур будет увеличиваться со временем, что приведет к возникновению индуцированного тока (по часовой стрелке, согласно закону Ленца).В какой-то момент \ (t \) поток через петлю определяется выражением:

\ [\ begin {align} \ Phi_B (t) & = \ vec B \ cdot \ vec A = BA = BLvt \ end {align} \]

, где мы выбрали \ (\ vec A \) параллельным вектору магнитного поля.

Поскольку мы уже использовали закон Ленца, чтобы доказать, что ток должен идти по часовой стрелке, мы можем использовать закон Фарадея для определения величины индуцированного напряжения и игнорировать отрицательный знак:

\ [\ begin {align} \ Delta V = \ frac {d \ Phi_B} {dt} = \ frac {d} {dt} BLvt = BLv \ end {align} \]

Предположим, что рельсы сверхпроводящие (не имеют сопротивления), а стержень имеет сопротивление \ (R \).2} {R} \ end {align} \]

Таким образом, стержень не может двигаться с постоянной скоростью сам по себе, иначе энергия будет производиться из ничего. К штанге должна прилагаться сила, чтобы она двигалась с постоянной скоростью.

Напомним, что на провод с током в магнитном поле действует сила магнитного поля. В этом случае стержень длиной \ (L \) переносит ток \ (I \) в магнитном поле \ (\ vec B \) (перпендикулярно току), так что сила, действующая на стержень выдает:

\ [\ начало {выровнено} \ vec F_B = I \ vec L \ times \ vec B \ end {выровнено} \]

и указывает налево (правило правой руки).2} {R} \ end {align} \]

, где мы предположили, что полоса движется в положительном направлении \ (x \). Это именно та скорость, с которой электрическая энергия рассеивается в баре! Другими словами, выполняя механическую работу со штангой, мы можем создать индуцированный ток, который будет рассеивать эту энергию с той же скоростью, с которой мы работаем. Мы можем преобразовывать механическую работу в электрическую энергию!

Наконец, также обратите внимание, что эта ситуация тесно связана с эффектом Холла, который представляет собой просто другой способ взглянуть на эту проблему.Рассмотрим электроны, которые находятся в стержне, поскольку стержень движется с постоянной скоростью вправо через магнитное поле (не обращайте внимания на наличие U-образной направляющей). Электроны будут испытывать магнитную силу, направленную вверх (в соответствии с направлением индуцированного тока, о котором говорилось выше). В конце концов, электроны накапливаются в верхней части стержня и начинают препятствовать накоплению там большего количества электронов, создавая электрическое поле \ (\ vec E \) в стержне. Условие равновесия состоит в том, что магнитная сила и электрическая сила имеют одинаковую величину (и противоположные направления):

\ [\ begin {Выровнено} qvB & = qE \\ E & = vB \ end {Выровнено} \]

(Холловская) разность потенциалов на стержне длиной \ (L \) с электрическим полем \ (E \) определяется выражением:

\ [\ begin {выравнивается} \ Delta V_ {Hall} = EL = vBL \ end {выравнивается} \]

, где мы предположили, что электрическое поле в стержне однородно.Эта разность потенциалов идентична той, которую мы вычислили по закону Фарадея. Рассмотрение этого примера как другого проявления эффекта Холла дает некоторое представление о том, что на самом деле происходит на микроскопическом уровне, когда индуцируется ток.

Генератор

Электрический генератор используется для создания переменного индуцированного напряжения / тока путем вращения катушки внутри постоянного и однородного магнитного поля. В этом случае ток индуцируется, потому что угол между магнитным полем и вектором элемента поверхности \ (d \ vec A \) изменяется со временем.

Рассмотрим одиночный виток провода с площадью \ (A \), который может вращаться в однородном и постоянном магнитном поле, \ (\ vec B \), как показано на рисунке \ (\ PageIndex {2} \).

Рисунок \ (\ PageIndex {2} \): петля из проволоки вращается в постоянном и однородном магнитном поле. В момент времени \ (t = 0 \) (левая панель) петля лежит в плоскости \ (yz \). Петля вращается вокруг оси \ (y \) с постоянной угловой скоростью \ (\ vec ω \). Через некоторое время t петля повернулась на угол \ (θ = ωt \) (правая панель, если смотреть сверху, если смотреть вниз на плоскость \ (xz \)).

В системе координат, показанной на рисунке \ (\ PageIndex {2} \), контур имеет постоянную угловую скорость \ (\ vec \ omega \) в положительном направлении \ (y \) и вращается вокруг ось \ (y \) (с началом в центре катушки). В момент времени \ (t = 0 \) (левая панель) петля лежит в плоскости \ (yz \), и мы выбираем вектор \ (\ vec A \) (используемый для вычисления потока), чтобы он находился в положительное направление \ (x \) в момент времени \ (t = 0 \). По мере вращения катушки будет вращаться вектор \ (\ vec A \), который легче визуализировать, чем катушку.В какой-то момент \ (t \) вектор \ (\ vec A \) составит угол \ (\ theta = \ omega t \) с осью \ (x \) (правая панель). Магнитное поле постоянно и в положительном направлении \ (x \), \ (\ vec B = B \ hat x \). То есть угол между вектором \ (\ vec A \) и магнитным полем \ (\ vec B \) будет равен \ (\ theta = \ omega t \).

В какой-то момент \ (t \) вектор \ (\ vec A \) задается следующим образом: \ [\ begin {align} \ vec A (t) = A (\ cos \ theta \ hat x — \ sin \ theta \ hat z) = A (\ cos (\ omega t) \ hat x — \ sin (\ omega t) \ hat z) \ конец {выровнено} \]

Мы можем вычислить поток магнитного поля через петлю в какой-то момент времени \ (t \): \ [\ begin {align} \ Phi_B (t) = \ vec B \ cdot \ vec A = (B \ hat x) \ cdot (\ cos (\ omega t) \ hat x — \ sin (\ omega t) \ hat z) = AB \ cos (\ omega t) \ end {align} \], где мы не использовали интеграл для потока, так как магнитное поле постоянно по площади контура.Индуцированное напряжение определяется законом Фарадея: \ [\ begin {align} \ Delta V = — \ frac {d \ Phi_B} {dt} = — \ frac {d} {dt} AB \ cos (\ omega t) = AB \ omega \ sin (\ omega t) \ end {align} \] Если генератор включает \ (N \) петель в катушке, то индуцированное напряжение определяется следующим образом: \ [\ begin {align} \ Delta V = NAB \ omega \ sin (\ omega t) \ end {align} \] Как вы можете видеть, напряжение колеблется со временем между \ (\ pm NAB \ omega \), что соответствует переменному напряжению. Кроме того, поскольку знак \ (\ Delta V \) меняется со временем (из-за синусоидальной функции), относительная ориентация между \ (\ vec A \) и магнитным дипольным моментом индуцированного тока также изменяется со временем. , показывая, что индуцированный ток в катушке меняет направление каждые пол-оборота (переменный ток).

Генераторы, вырабатывающие переменное напряжение, которое мы находим в наших розетках, работают по тому же принципу. Например, в гидроэлектрической плотине давление воды с высоты плотины используется для проталкивания воды через турбину (по сути пропеллер), которая вращает набор катушек внутри сильного постоянного магнита. Различные элементы управления позволяют регулировать частоту вращения турбины для выработки переменного тока желаемой частоты (\ (50 \ text {Hz} \) в большинстве стран мира, \ (60 \ text {Hz} \) в Северная Америка и несколько других стран).

Поскольку генератор вырабатывает ток, который может рассеивать электрическую энергию, необходимо выполнять работу, чтобы катушка в генераторе оставалась вращающейся. При вращении катушки в катушке индуцируется ток. Ток в круговой петле, погруженной в магнитное поле, будет испытывать крутящий момент \ (\ vec \ tau \), задаваемый следующим образом: \ [\ begin {align} \ vec \ tau = \ vec \ mu \ times \ vec B \ end {align} \] где \ (\ vec \ mu \) — магнитный дипольный момент катушки с индуцированным током, \ (I \). Если ток из катушки рассеивает свою энергию в системе с сопротивлением \ (R \), то ток в катушке определяется законом Ома:

\ [\ begin {align} I = \ frac {\ Delta V} {R} = \ frac {NAB \ omega \ sin (\ omega t)} {R} \ end {align} \]

Магнитный момент \ (\ vec \ mu \) для тока в катушке определяется выражением:

\ [\ begin {align} \ vec \ mu & = I \ vec A = \ frac {NAB \ omega \ sin (\ omega t)} {R} (A (\ cos (\ omega t) \ hat x — \ sin (\ omega t) \ hat z)) \\ & = \ frac {NA ^ 2B \ omega \ sin (\ omega t)} {R} (\ cos (\ omega t) \ hat x — \ sin ( \ omega t) \ hat z) \ end {align} \]

Таким образом, крутящий момент, создаваемый магнитным полем на катушке с индуцированным током, определяется выражением:

\ [\ begin {align} \ vec \ tau & = \ vec \ mu \ times \ vec B = \ left (\ frac {NA ^ 2B \ omega \ sin (\ omega t)} {R} (\ cos ( \ omega t) \ hat x — \ sin (\ omega t) \ hat z) \ right) \ times (B \ hat x) \\ & = \ frac {NA ^ 2B ^ 2 \ omega \ sin (\ omega t )} {R} (\ cos \ omega (t) (\ hat x \ times \ hat x) — \ sin (\ omega t) (\ hat z \ times \ hat x)) \\ & = — \ frac { NA ^ 2B ^ 2 \ omega \ sin ^ 2 (\ omega t)} {R} \ hat y \ end {align} \]

Обратите внимание, что крутящий момент, действующий на контур, всегда имеет отрицательное направление \ (y \), поскольку каждый член крутящего момента либо строго положителен (\ (N, R \)), либо возведен в квадрат (\ (\ sin ^ 2 (\ омега т) \)).Таким образом, крутящий момент, прилагаемый магнитным полем к катушке, всегда направлен в противоположном направлении вращения (напомним, что катушка имеет угловую скорость в положительном направлении \ (y \)). Иногда это называют «противодействующим моментом». Если мы хотим, чтобы катушка поддерживала постоянную угловую скорость, мы должны приложить крутящий момент в положительном направлении \ (y \), чтобы противодействовать крутящему моменту от магнитного поля. Обратите внимание, что крутящий момент, который мы должны приложить, чтобы катушка вращалась с постоянной угловой скоростью, не является постоянным во времени (но всегда в одном и том же направлении).

Вы можете легко проверить, что работа, которую вы должны выполнить, приложив крутящий момент, такая же, как электрическая мощность, рассеиваемая током в резисторе \ (R \). Таким образом, генератор представляет собой устройство для преобразования механической работы в электрическую энергию (в частности, с переменным током).

Указания по безопасной работе с кабельной системой при наведенном напряжении или токе

участников

Организатор (DK)
U.С. ГУДМУНДСДОТТИР

Секретарь (DK)
К.SCHULTZ PEDERSEN

A. BURGOS MELGUIZO (ES), A. BARCLAY (GB), G. BUCEA (AU), G. SVEJDA (AT), J. MATALLANA (NO), L. GUIZZO (IT), M. FREILINGER (CH) , M. CABAU (Франция), M. OLTMANS (Нидерланды), M. KLEIN (AT), V. DUBICKAS (SE), Y. LE ROY (BE)

Члены-корреспонденты: C. MOUYCHARD (FR), D.МАЙЯ ДЕ АЛМЕЙДА (Бразилия), У. ХУАНГ (Великобритания)

Фон

В течение нескольких этапов жизненного цикла кабельной системы (установка / обслуживание / тестирование / модернизация / удаление) может возникнуть необходимость работать под напряжением или током, индуцированным системой под напряжением: Например:

  • Во время протягивания или прокладки:
  • Во время операций соединения в процессе установки
  • При проверке или обслуживании блоков связи
  • Во время ремонта кабеля после неисправности
  • В процессе обновления существующей схемы
  • При удалении кабеля для утилизации в конце это жизнь.

Поскольку могут возникнуть опасные условия, важно предоставить всем сторонам, которые могут быть вовлечены (коммунальные предприятия, производители, установщики, испытательные институты…), инструкции по безопасной работе с кабельными системами, включая четкую терминологию.

Область применения

Техническая брошюра представляет собой исчерпывающее руководство, в котором рассматриваются все темы, связанные с работой с наведенными напряжениями или токами на наземных кабелях, а также некоторые из них, касающиеся морских кабелей. Руководство охватывает:

  1. Экструдированные кабельные системы или кабельные системы внахлест
  2. ВН, но также кабели СН и даже НН переменного тока, когда они являются частью схемы подключения,
  3. Постоянные или аварийные состояния (напряжения кабельной системы при неисправности сети)
  4. Методы для рассчитать наведенные напряжения и / или токи в различных возможных конфигурациях (включая ЭДС или магнитный эффект от кабелей или воздушных линий, установленных поблизости)
  5. Защитное оборудование (перчатки, системы заземления….), необходимые для обеспечения безопасных условий работы.
  6. Все возможные виды работ, такие как укладка, стыковка, заделка и работа на ящиках ссылок.

В документе не приводится подробный анализ эффектов захваченных зарядов и «обратного напряжения» кабелей HVDC (которые упоминаются только кратко) или ЭДС. Эти темы не входят в объем работ.

Рисунок 1 — Пример одножильного кабеля высокого / сверхвысокого напряжения

Содержимое

В Технической брошюре описаны различные типы кабельной разводки и монтажа.Рассмотрены туннели, открытая траншея, каналы, ГНБ, вспашки и морские сооружения. Для различных настроек указаны специальные меры безопасности и соответствующее оборудование.
В данной технической брошюре упоминаются риски, связанные только с наведенным напряжением в кабельных системах; он не оценивает и не исследует другие возможные проблемы безопасности, связанные с другими опасностями в кабельных системах, такими как механические нагрузки и т. д.

В главе 1, , Техническая брошюра начинается с обсуждения и определения явления наведенного напряжения, которое может иметь несколько источников.
С целью определения того, что является опасной средой из-за наведенных напряжений или токов, необходимо определить пределы напряжений прикосновения. Поскольку это варьируется от страны к стране и существует несколько различных стандартов, обсуждающих эту тему, задача Рабочей группы B1.44 включала углубленную оценку этих стандартов и руководящих принципов и тщательное рассмотрение того, что следует рекомендовать и включен в соответствующий международный стандарт.

Рисунок 2 — Временные / текущие кривые воздействия переменного тока на тело человека [IEC 60479]

Механизмы наведенного напряжения

Основной принцип, подчеркнутый в руководстве, заключается в том, что перед началом любых работ с системами силовых кабелей настоятельно рекомендуется провести анализ рисков и расчет возможных наведенных напряжений.Предлагаемый анализ рисков объясняется в Принципах безопасной работы в , Глава 2 . Приведены и подробно описаны три различных принципа безопасной работы: работа с заземлением без токов (это рекомендуемый метод), работа с заземлением с током и работа с изолированной работой.

В Глава 3 дается подробное описание безопасных рабочих процедур для нескольких этапов работы с кабельной системой: прокладка / протягивание, резка, соединение, заделка, испытание, снятие. Напоминаем, что все эти работы должны выполняться только квалифицированным и аттестованным персоналом.

Глава 4 предоставляет методологию расчета трех различных типов наведенных напряжений. Некоторые реальные примеры кабельных систем, испытывающих наведенные напряжения из разных стран, подробно описаны в приложениях. Выводы и рекомендации приведены в главе 5.

Механизмы наведенного напряжения

Существует несколько типов наведенных напряжений, которые могут влиять на потенциал жилы кабеля, экрана и металлической оболочки. Согласно предыдущим исследованиям СИГРЭ, различные типы можно разделить на три категории в зависимости от того, как они связаны от источника электрического тока к объекту индукции.

  • индуктивная связь
  • емкостная связь
  • проводящая связь

При работе с наведенными напряжениями необходимо учитывать все три проблемы. Влияние каждой муфты зависит от наличия других электрических систем вокруг кабеля и от того, как кабельная система заземлена.

Индуктивная муфта

Изолированный провод, проходящий вблизи линии электропередачи или кабеля, либо на всем протяжении, либо на части его маршрута, подвергается воздействию напряжений, вызванных магнитной связью.

Рисунок 3 — Пример сети с параллельными линиями

Связь возникает из-за изменяющегося во времени магнитного потока от источника индукции, связанного с цепью, образованной проводниками объекта индукции.

Ток в находящемся поблизости кабеле или воздушной линии (ВЛ) в эксплуатации, будь то в нормальных условиях или из-за неисправности, будет индуцировать напряжение в продольном направлении на изолированном проводе (который либо устанавливается, ремонтируется или обслуживается).Если расстояние невелико, и особенно если в работающей системе присутствует асимметричный ток (например, ток короткого замыкания из-за неисправности), индуцированное напряжение может достигать нескольких киловольт. Это индуцированное напряжение будет продольно перемещаться по всем металлическим частям кабеля, находящимся в рабочем или нерабочем состоянии, то есть к броне, металлической оболочке, проводнику и параллельным заземляющим проводам (таким как проводники заземления, ECC).
Величина индуцированного напряжения из-за индуктивной связи зависит от тока в системе источника, расстояния между двумя системами и длины параллельности двух систем. Следует отметить, что величина индуцированного напряжения индуктивной связи не зависит от уровня напряжения системы.

Емкостная муфта

Емкостная связь является результатом электрического поля вокруг энергосистемы, находящейся под напряжением. Электрическое поле связано с емкостью между источником и любым незаземленным проводящим объектом, подвергающимся воздействию поля. Вместе с емкостью между незаземленным объектом и землей образуется емкостной делитель напряжения, так что для источника переменного напряжения часть напряжения источника появляется на объекте.
Следовательно, воздушные линии или шины подстанции могут воздействовать только на изолированные кабели, не экранированные ни землей, ни заземленным экраном. Это влияние происходит как при нормальной работе, так и при неисправности.
Напротив, подземные и морские кабели не могут создавать емкостную связь, поскольку электрическое поле содержится внутри экранированной изоляции.

Между изолированным проводом кабеля и землей возникают напряжения промышленной частоты; их величина зависит в основном от уровня напряжения ВЛ, расстояния между ВЛ и изолированным проводом, а также от условий эксплуатации ВЛ (нормальная работа или неисправности).Следует отметить, что величина емкостной связи не зависит от тока, протекающего в системах.
Всякий раз, когда кабель остается отключенным, металлическая оболочка и проводник с сердечником должны быть зашунтированы и локально заземлены, чтобы предотвратить постоянное напряжение. Следует отметить, что только часть кабельной системы должна подвергаться воздействию электрического поля для воздействия на весь кабель.

Рисунок 4 — Упрощенная схема емкостной связи

Муфта токопроводящая

Токи заземления (например, обратные токи короткого замыкания), протекающие через полное сопротивление заземления системы заземления (например, опоры, подстанции, соединительных ячеек или электростанции), вызывают повышение потенциала земли (EPR) системы заземления по отношению к любому удаленная земля.Проводники, подключенные к этой системе заземления, могут передавать повышение потенциала на любую другую систему заземления, к которой они могут быть подключены, которые могут находиться на расстоянии многих километров.

Кабельные системы часто соединяют две разные системы заземления (например, между разными подстанциями). В таких ситуациях следует рассмотреть возможность передачи РОП. Ситуация уже существует при нормальной работе высоковольтных систем (где EPR создается индуцированными токами или блуждающими токами) и усугубляется, когда есть неисправность в одной части системы, которая вызывает EPR на месте.Кабель передаст это повышение потенциала земли в другое место, имеющее другой потенциал земли. Следовательно, может быть разница в напряжении между двумя «заземленными» объектами, один из которых заземлен локально, а другой — удаленно.

Рисунок 5 — Кабельная система между различными системами заземления

Принципы безопасной работы

При установке новой кабельной цепи или при работе с существующей кабельной системой может возникнуть неминуемая угроза безопасности из-за опасного индуцированного или передаваемого напряжения и / или сильных циркулирующих токов, даже если кабель, с которым предстоит работать, отсоединен и изолирован от электрической системы. и заземлен.

Важно знать, что, хотя работа с изолированным кабелем, находящимся в нерабочем состоянии, при нормальных условиях работы системы / сети может быть безопасной, опасные напряжения могут возникать непредсказуемо в любое время из-за внешних неисправностей в электрической системе или из-за переключение, молния или другие скачки напряжения.

В результате, крайне важно, чтобы при планировании работ под воздействием наведенных напряжений разработанные рабочие процедуры должны включать оценку опасностей безопасности (электрические риски) и обеспечивать наиболее подходящие безопасные условия труда с учетом трех основных безопасных рабочих принципов:

  • заземленный, работающий с токами
  • заземленный, работающий без токов
  • рабочий, изолированный (с индуцированным или постоянным напряжением)

Для каждого принципа работы представлены подробные иллюстрации и соответствующие инструменты.

Рисунок 6а — Пример рабочего места и средств защиты для изолированных рабочих Рисунок 6b — Принципы подготовки рабочего места для изолированной работы Рисунок 7a — Пример рабочего места с надлежащей эквипотенциальностью и заземлением на металлической оболочке с использованием заземляющих проводов Рисунок 7b — Пример рабочего места, где подготовлено заземление для рабочих процедур

Перед выполнением любых работ на электроустановке необходимо провести оценку электрических рисков.В этой оценке необходимо указать, как должна выполняться операция и какие меры безопасности и меры предосторожности следует применять для обеспечения безопасности.

Если кабельная система полностью или частично параллельна другому подземному кабелю (UGC) или ВЛ, находящемуся в эксплуатации, при транспортировке необходимо уделять особое внимание.

Оценка наведенных напряжений и потенциального подъема почвы может быть получена аналитически с помощью уравнений, подробно описанных в главе 4, или путем моделирования. Электрические параметры, ток и напряжение должны быть рассчитаны как в установившемся, так и в аварийном режиме (короткое замыкание, гроза и импульсные перенапряжения).Следует учитывать несколько случаев в зависимости от типа аварийных событий, которые могут произойти в соседних линиях электропередач. Для проверки модели рекомендуется выполнить некоторые измерения на месте. Если невозможно точно рассчитать индуцированные напряжения из-за неточной информации о внешних факторах или неизвестных факторах, для расчета следует принять наихудший сценарий.

Рисунок 8 — Принципы магнитной муфты

В случае наведенных напряжений или EPR выше максимально допустимого наведенного напряжения, анализ рисков должен также включать рекомендации по безопасному выполнению работ.В этой рекомендации должен быть указан максимальный размер возможных сопротивлений заземления для снижения наведенных напряжений, номинальный ток для заземляющих соединений и надлежащее пошаговое описание физических работ. Для выполнения работ по соединению и заделке следует запросить надлежащие рабочие процедуры, обеспечивающие безопасность и заземление, у лица, ответственного за работу. Перед началом любых работ важно обеспечить надлежащую эквипотенциальность и заземление. Если невозможно работать с эквипотенциальностью или если есть потребность в изолированной работе, тип и уровень изолированного инструмента должны быть описаны в анализе риска.

Выводы и рекомендации

В этой технической брошюре представлены и обсуждаются некоторые аспекты наведенных напряжений в силовых кабельных системах. Были рассмотрены три типа наведенных напряжений; (i) — индуктивная связь, (ii) — емкостная связь и (iii) — кондуктивная связь (EPR).
В данной технической брошюре основное внимание уделяется тому, как рассчитать наведенные напряжения на кабеле, над которым нужно работать, как спланировать работы в случае индуцированных напряжений и как продолжить работу в ситуациях, когда существует риск возникновения опасного напряжения или токи.

Предполагая, что существует риск индукции, вводятся два основных принципа безопасной работы; т.е. «Заземленные работы» и «Изолированные работы». Для условий работы с заземлением можно применять либо заземленную работу без токов, где есть только заземление, выполненное локально на рабочем месте, либо применять заземленные условия работы с токами, когда кабель также подключается к земле на кабеле. далеко концы. В обоих случаях сначала необходимо обеспечить эквипотенциальность в рабочей зоне .
В Технической брошюре рассматриваются все принципы, но в брошюре рекомендуются заземленные условия работы без токов в качестве основного решения для обеспечения безопасной работы там, где существует риск наведенных напряжений, вызванных индуктивной связью, емкостной связью или EPR.
В любом случае, все работы по установке при наведенном напряжении всегда должны выполняться только квалифицированным и сертифицированным персоналом и в соответствии с местными законодательными требованиями, если таковые имеются.
В приложения к этой технической брошюре включены несколько примеров того, как выполнять расчеты наведенного напряжения, и несколько реальных примеров из разных стран.

Одним из важных выводов, которые были выявлены, является отсутствие четкого руководства или соответствующего стандарта для оценки максимально допустимого напряжения прикосновения для кабельных систем за пределами подстанций. В результате в этой технической брошюре цитируется несколько стандартов и руководств, которые связаны и используются в разных странах. Кроме того, в эту Техническую брошюру включена рекомендация для МЭК обновить существующий стандарт МЭК 61936, в котором указаны пределы напряжения прикосновения как функция времени зазора для кабельных систем на подстанциях.Рекомендация упоминается в главе 1 и поясняется далее в первом приложении.

Другие технические брошюры

Индуцированный потенциал и эффект генератора — Индуцированный потенциал и трансформаторы — Eduqas — Редакция GCSE Physics (Single Science) — Eduqas

Для протекания электрического тока в цепи требуется разность потенциалов или напряжение.

Создание разности потенциалов

Разность потенциалов может индуцироваться (создаваться) в проводнике при движении между проводником и магнитным полем. Это может происходить двумя разными способами:

  • катушка провода перемещается в магнитном поле
  • магнит перемещается в катушку провода

Это называется электромагнитной индукцией и часто упоминается как эффект генератора.

Индуцированное напряжение создает индуцированный ток, если проводник включен по замкнутой цепи.Как и все токи, индуцированный ток создает вокруг себя магнитное поле. Обратите внимание, что это магнитное поле противодействует первоначальному изменению. Например, если магнит помещается в катушку с проволокой, индуцированное магнитное поле имеет тенденцию отталкивать магнит обратно из катушки. Этот эффект возникает независимо от того, перемещается ли магнит в катушку или катушка перемещается вокруг магнита.

Факторы, влияющие на наведенный потенциал

Направление наведенной разности потенциалов или наведенного тока зависит от направления движения.Ток меняется на противоположное, когда:

  • магнит перемещается из катушки
  • другой полюс магнита перемещается в катушку

Изображения показывают, как это работает.

1. Стержневой магнит находится за пределами катушки с проволокой, подключенной к амперметру, показывающему отсутствие тока

2. Магнит перемещается в катушку с проволокой, и амперметр регистрирует положительный поток тока

3. Магнит неподвижен в катушке провод. Нет тока

4.Магнит выходит из катушки с проволокой, и амперметр регистрирует отрицательный ток.

Индуцированная разность потенциалов или индуцированный ток увеличиваются, если:

  • скорость движения увеличивается
  • увеличивается напряженность магнитного поля
  • число витков на катушке увеличивается

Это обратное или индуцированное напряжение?

Вы собираетесь проверить отсутствие напряжения и прошли процедуру блокировки / маркировки.Вы носите соответствующие средства индивидуальной защиты. У вас есть подходящий тестер напряжения, и вы знаете, как им пользоваться. Прикоснувшись щупами к цепи, вы получите напряжение там, где его не должно быть! Что происходит?

1. Возможно, вы выбрали не то оборудование. Уж точно не ты! Это настолько большая проблема, что NFPA 70E включил новую статью об этом в редакцию 2009 года в статье 130.7 (E), Методы оповещения. Он гласит: «(4) Двойное оборудование.Если работа, выполняемая на оборудовании, которое обесточено и находится в электрически безопасном состоянии, существует в рабочей зоне с другим находящимся под напряжением оборудованием, аналогичным по размеру, форме и конструкции, один из методов изменения в 130.7 (E) (1), (2) или (3) должны использоваться для предотвращения доступа сотрудника к похожему оборудованию ».

2. Неужели он выключен? Если прерыватель или предохранитель, питающий нашу цепь, не имеет четкой маркировки или если сработал автоматический выключатель в литом корпусе, происходят шокирующие вещи! Я несколько раз бывал от «сработавшего» автоматического выключателя в литом корпусе только для того, чтобы обнаруживать, что контакты не полностью размыкаются.В них не было бы тока, но они точно несли бы напряжение! Перед тем, как приступить к работе, всегда устанавливайте сработавший выключатель в положение полного выключения.

3. Могут присутствовать индуцированные или «фантомные» напряжения. Многие думают, что индуцированные напряжения возникают только на высоковольтных подстанциях вне помещения. Хотя это самая большая опасность из-за наведенных напряжений, низковольтные цепи, проложенные в кабельных лотках, могут также индуцировать напряжение в обесточенных кабелях, которые находятся в том же кабельном лотке (см. Рисунок 1). Применение статического заземления к этой цепи без проблем рассеяло бы напряжение, поскольку индуцированное напряжение не имеет способности к току короткого замыкания.

Рис. 1. Сценарий наведенного напряжения низкого напряжения

4. Может быть откормлен. Управляющие силовые трансформаторы (CPT), сигнальные лампы и «посторонние» цепи (исходящие от другой панели или области) могут быть виноваты. Применение статического заземления к цепи с обратным питанием может вызвать искрение, что небезопасно.

Напряжение обратной связи

Часто обратные напряжения и индуцированные напряжения могут быть очень похожими.Индуцированные напряжения обычно намного ниже номинального напряжения схемы, но обратные токи могут находиться в том же диапазоне напряжений, что и наведенные напряжения. Поскольку заземлять обратное питание небезопасно, что мы можем сделать?

Обратные напряжения — это напряжения, которые часто возникают из другой цепи или части оборудования, но «подаются» через световые индикаторы, управляющие силовые трансформаторы или даже резисторы в оборудовании. Эти напряжения обычно меньше номинального напряжения цепи и могут быть примерно такими же, как индуцированные напряжения.

Иногда бывает сложно отличить обратное или индуцированное напряжение. Если индуцированное напряжение подключено к земле, источник генерации (тока) отсутствует, и напряжение будет рассеиваться. Обратное напряжение, даже если оно ниже номинального, имеет источник, питающее его, и при подключении к земле возникает дуга.

Приборы для испытаний с низким сопротивлением и высоким сопротивлением

Решение состоит в том, чтобы использовать комбинацию инструментов тестирования, чтобы определить, является ли это резервным или индуцированным, а затем проверить первоначальные результаты.

Качественные тестеры напряжения обычно имеют высокий входной импеданс. Я понял ценность этого, когда тестировал чиллер на 9000 тонн, у которого периодически возникала проблема. Я подключил испытательный зонд к одной стороне катушки, и когда я коснулся земли другим зондом, катушка замкнулась, отключив чиллер. Это не был момент для карьерного роста.

Входной импеданс измерителя, который я использовал, составлял всего несколько тысяч Ом. Когда я подключил катушку под напряжением к земле, через измеритель протекло достаточно тока для работы катушки.Измеритель с высоким входным импедансом не пропустит через измеритель достаточный ток, чтобы катушка заработала. Я взял свой недорогой мультиметр с низким входным сопротивлением домой и купил устройство хорошего качества с высоким входным сопротивлением.

Итак, после первого измерения напряжения с помощью стандартного вольтметра с высоким входным импедансом, используйте измеритель с опцией низкого импеданса, такой как Fluke 117 или 289. Эти измерители предлагают функции как с высоким, так и с низким входным импедансом. Если напряжение индуцировано, низкоомный вход должен рассеивать напряжение после того, как он подключен к земле.

Используя низковольтный бесконтактный тестер, измерьте длину тестируемой цепи, пока еще подключен низкоомный тестер напряжения. На рисунке 2 показаны конечные показания; нет напряжения, показываемого бесконтактным тестером, и никакого напряжения, отображаемого тестером с низким входным сопротивлением.

Рисунок 2. Индикация наведенного напряжения

Рисунок 3. Индикация обратного напряжения

Если тестер напряжения с низким входным импедансом измеряет напряжение, как на рисунке 3, даже если оно может составлять всего несколько вольт, а бесконтактный тестер показывает наличие напряжения, напряжение в цепи, вероятно, является обратным током и его необходимо определить. прежде чем продолжить.Заземление этой цепи приведет к дуговой сварке!

Измеритель двойного импеданса идеально подходит для этого теста — лучше, чем носить с собой два отдельных измерителя или делать небезопасное измерение.

Резюме

Если вы обнаружите цепь, которая показывает напряжение, хотя его не должно быть, будьте осторожны, что делать дальше. Создание дуги небезопасно и может привести к увольнению или даже хуже. Быть безопасным. Определите, индуцируется ли напряжение расположенными поблизости кабелями, находящимися под напряжением, или оно создается из неизвестного источника.

Об авторе:
Джим Уайт — директор по обучению в Shermco Industries в Ирвинге, штат Техас, и технический специалист уровня IV NETA. Джим представляет NETA в комитетах NFPA 70E и B, а также в Рабочей группе по опасностям дугового разряда и является председателем семинара по электробезопасности IEEE 2008 года.

29 Свода федеральных правил, § 1926.964 — Работы на воздушных линиях и в прямом эфире голыми руками. | CFR | Закон США

§ 1926.964 Работа над воздушными линиями и живыми линиями голыми руками.

(а) Общие —

(1) Заявление. В этом разделе представлены дополнительные требования к работам, выполняемым на воздушных линиях и оборудовании или вблизи них, а также к работам без экипажа под напряжением.

(2) Проверка конструкции перед подъемом. Прежде чем разрешить сотрудникам подвергать возвышенные конструкции, такие как столбы или башни, таким нагрузкам, которые могут возникнуть при подъеме или установке или снятии оборудования, работодатель должен убедиться, что конструкции способны выдерживать дополнительные или несбалансированные нагрузки.Если столб или другая конструкция не может выдержать ожидаемые нагрузки, работодатель должен закрепить или иным образом поддержать столб или конструкцию, чтобы предотвратить поломку.

Примечание к пункту (а) (2):

Приложение D к этому подразделу содержит методы испытаний, которые работодатели могут использовать для определения того, способен ли деревянный столб выдерживать силы, создаваемые работником, взбирающимся на столб. Этот параграф также требует, чтобы работодатель удостоверился, что столб может выдержать все другие силы, создаваемые работой, которую будут выполнять сотрудники.

(3) Установка и перемещение столбов.

(i) Когда полюс устанавливается, перемещается или снимается рядом с открытым электрическим проводом, находящимся под напряжением, полюс может не соприкасаться с проводником.

(ii) Когда столб устанавливается, перемещается или снимается рядом с незащищенным воздушным проводом под напряжением, работодатель должен обеспечить, чтобы каждый работник носил электрозащитное оборудование или использовал изолированные устройства при работе со столбом и чтобы ни один сотрудник не касался столба неизолированными частями. его или ее тела.

(iii) Для защиты сотрудников от падения в ямы, используемые для установки столбов, работодатель должен физически охранять ямы или обеспечивать присутствие сотрудников в ямах всякий раз, когда кто-либо работает поблизости.

(б) Монтаж и демонтаж ВЛ. Следующие положения применяются к установке и удалению воздушных проводов или кабеля (воздушных линий).

(1) Метод натяжения нити. Когда линии, которые устанавливают или удаляют сотрудники, могут соприкасаться с частями под напряжением, работодатель должен использовать метод натяжения, барьеры или другие эквивалентные меры, чтобы свести к минимуму вероятность того, что проводники и кабели, устанавливаемые или удаляемые сотрудниками, будут контактировать с линиями электропередачи или оборудованием, находящимися под напряжением. .

(2) Проводники, кабели, тяговое и натяжное оборудование. Для проводов, кабелей, а также тягового и натяжного оборудования работодатель должен обеспечить защитные меры, требуемые § 1926.959 (d) (3), когда сотрудники устанавливают или удаляют провод или кабель достаточно близко к проводникам под напряжением, чтобы любой из следующих сбоев мог подать питание на тянущее или натяжное оборудование, проводник или кабель, устанавливаемый или удаляемый:

(i) Отказ тягового или натяжного оборудования,

(ii) Отказ протягиваемого проводника или кабеля, или

(iii) Выход из строя ранее установленных линий или оборудования.

(3) Отключить функцию автоматического повторного включения. Если проводники, на которых сотрудники устанавливают или удаляют перекрестные проводники под напряжением, напряжение которых превышает 600 вольт, и если это позволяет конструкция устройств прерывания цепи, защищающих линии, работодатель должен вывести из строя функцию автоматического повторного включения этих устройств.

(4) Наведенное напряжение.

(i) Перед тем, как сотрудники установят линии, параллельные существующим линиям под напряжением, работодатель должен определить приблизительное напряжение, которое будет индуцировано в новых линиях, или работа должна продолжаться, исходя из предположения, что индуцированное напряжение является опасным.

(ii) Если работодатель не может продемонстрировать, что линии, которые устанавливают работники, не подвержены наведению опасного напряжения, или если линии не рассматриваются как находящиеся под напряжением, в таких местах должны быть размещены временные защитные заземления и организованы таким образом. то, что работодатель может продемонстрировать, предотвратит воздействие на каждого работника опасной разницы в электрическом потенциале.

Примечание к пункту (b) (4) (ii):

Приложение C к этому подразделу содержит инструкции по защите сотрудников от опасной разницы в электрическом потенциале в соответствии с требованиями этого параграфа.

Примечание к пункту (b) (4):

Если работодатель не принимает мер предосторожности для защиты сотрудников от опасностей, связанных с непроизвольной реакцией на поражение электрическим током, существует опасность, если индуцированное напряжение достаточно, чтобы пропустить ток в 1 миллиампер через резистор на 500 Ом. Если работодатель защищает сотрудников от травм из-за непроизвольной реакции на поражение электрическим током, существует опасность, если результирующий ток будет более 6 миллиампер.

(5) Условия безопасной эксплуатации.Оборудование для работы с мотовилами, включая тяговые и натяжные устройства, должно быть в безопасном рабочем состоянии, выровнено и выровнено.

(6) Грузоподъемность. Работодатель должен гарантировать, что сотрудники не превышают допустимую нагрузку на натяжные тросы, тяговые тросы, зажимы для проводов, силовое оборудование и аксессуары, такелаж и подъемники.

(7) Неисправные тяговые стропы. Работодатель обязуется отремонтировать или заменить неисправные тяговые тросы и принадлежности.

(8) Зажимы для проводов.Работодатель должен гарантировать, что работники не используют зажимы для проводов на тросе, если производитель специально не разработал зажим для этого применения.

(9) Связь. Работодатель должен обеспечить, чтобы сотрудники поддерживали надежную связь с помощью двусторонней радиосвязи или других эквивалентных средств между тендером катушки и оператором буксирной установки.

(10) Эксплуатация тягача. Сотрудники могут использовать буксирную установку только тогда, когда это безопасно.

(11) Работы по накладным работам.Пока устройство с механическим приводом тянет провод или тяговую линию, а проводник или тяговая линия находится в движении, работодатель должен обеспечить, чтобы сотрудники не находились непосредственно под воздушными операциями или на траверсе, за исключением случаев, когда служащим необходимо направлять натяжку. носок или доска поверх или сквозь натяжной шкив.

(c) Работа голыми руками в режиме реального времени. В дополнение к другим применимым положениям, содержащимся в этом подразделе, к работе без экипажа в прямом эфире применяются следующие требования:

(1) Обучение.Перед тем, как работник использует или контролирует использование техники без подключения к сети под напряжением в цепях под напряжением, работодатель должен убедиться, что работник прошел обучение в соответствии с § 1926.950 (b) по технике и требованиям безопасности параграфа (c) настоящего документа. раздел.

(2) Существующие условия. Перед тем, как какой-либо сотрудник использует методику без использования проводов под напряжением на проводниках или частях высокого напряжения под напряжением, работодатель должен удостовериться в следующей информации в дополнение к информации о других существующих условиях, требуемых § 1926.950 (г):

(i) Номинальное напряжение цепи, в которой сотрудники будут выполнять работу,

(ii) зазоры на землю для линий и других частей, находящихся под напряжением, на которых сотрудники будут выполнять работу, и

(iii) Ограничения напряжения оборудования, которое будут использовать сотрудники.

(3) Изолированные инструменты и оборудование.

(i) Работодатель должен обеспечить, чтобы изолированное оборудование, изолированные инструменты, а также воздушные устройства и платформы, используемые сотрудниками, были спроектированы, испытаны и изготовлены для работы без экипажа под напряжением.

(ii) Работодатель должен следить за тем, чтобы работники содержали инструменты и оборудование в чистоте и сухости во время их использования.

(4) Отключить функцию автоматического повторного включения. Работодатель должен вывести из строя функцию автоматического повторного включения устройств прерывания цепи, защищающих линии, если конструкция этих устройств позволяет.

(5) Неблагоприятные погодные условия. Работодатель должен гарантировать, что работники не выполняют работу, когда неблагоприятные погодные условия могут сделать работу опасной, даже после того, как работодатель применяет методы работы, требуемые данным подразделом.Кроме того, сотрудники не могут выполнять работу, когда ветер сокращает межфазные промежутки или межфазные расстояния до земли на рабочем месте ниже минимальных расстояний подхода, указанных в параграфе (c) (13) этого раздела, если только изолирующие ограждения не закрывают заземленные объекты и другие линии и оборудование.

Примечание к пункту (c) (5):

Грозы поблизости, сильные ветры, снежные бури и ледяные бури являются примерами неблагоприятных погодных условий, которые делают работу без экипажа под напряжением слишком опасной для безопасного выполнения даже после того, как работодатель применяет методы работы, требуемые этим подразделом.

(6) Гильзы ковша и электростатическая защита. Работодатель должен обеспечить и гарантировать, что сотрудники используют токопроводящую прокладку ковша или другое токопроводящее устройство для соединения изолированного воздушного устройства с линией или оборудованием, находящимся под напряжением.

(i) Работник должен быть подключен к футеровке ведра или другому токопроводящему устройству с помощью токопроводящей обуви, зажимов для ног или других средств.

(ii) Если разница потенциалов на рабочем месте представляет опасность для работников, работодатель должен обеспечить электростатическое экранирование, рассчитанное на рабочее напряжение.

(7) Привязка работника к части, находящейся под напряжением. Работодатель должен гарантировать, что до того, как работник коснется части, находящейся под напряжением, работник прикрепит токопроводящую прокладку ковша или другое токопроводящее устройство к проводнику под напряжением посредством положительного соединения. Это соединение должно оставаться присоединенным к проводнику под напряжением до тех пор, пока работник не завершит работу в цепи под напряжением.

(8) Органы управления подъемником. Подъемники, используемые для работы без экипажа на тросе, должны иметь следующие двойные органы управления (нижний и верхний):

(i) Верхние органы управления должны находиться в пределах досягаемости работника в ковше.На подъемнике с двумя ковшами доступ к органам управления должен находиться в пределах досягаемости обоих ковшей.

(ii) Нижний набор органов управления должен находиться рядом с основанием стрелы и должен быть спроектирован так, чтобы они могли блокировать работу оборудования в любое время.

(9) Работа нижних органов управления. Органы управления нижним (с уровня земли) подъемником нельзя использовать с работником, находящимся в лифте, за исключением экстренных случаев.

(10) Проверьте органы управления. Работодатель должен гарантировать, что перед тем, как сотрудники поднимут канатный подъемник в рабочее положение, сотрудники проверят все средства управления (уровень земли и ковш), чтобы определить, находятся ли они в надлежащем рабочем состоянии.

(11) Кузов автовышки. Работодатель должен обеспечить, чтобы перед тем, как работники поднимут стрелу подъемника, работники заземлили кузов грузовика или забаррикадировали кузов грузовика и относились к нему как к находящемуся под напряжением.

(12) Испытание на ток стрелы. Работодатель должен гарантировать, что сотрудники проводят испытание на силу тока стрелы каждый день перед началом работы, каждый раз в течение дня, когда они сталкиваются с более высоким напряжением, и когда изменение условий указывает на необходимость дополнительного испытания.

(i) Это испытание должно состоять из приведения ковша в контакт с источником под напряжением, равным возникающему напряжению, в течение минимум 3 минут.

(ii) Ток утечки не должен превышать 1 микроампер на киловольт номинального напряжения между фазой и землей.

(iii) Работодатель должен немедленно прекратить работу на канатном подъемнике, если есть какие-либо признаки неисправности в оборудовании.

(13) Минимальная дистанция подхода. Работодатель должен гарантировать, что работники соблюдают минимальную дистанцию ​​подхода, установленную работодателем в соответствии с § 1926.960 (c) (1) (i), от всех заземленных объектов, а также от линий и оборудования с потенциалом, отличным от того, к которому подключено неизолированное оборудование под напряжением, если только изоляционные ограждения не закрывают такие заземленные объекты и другие линии и оборудование.

(14) Приближение, выход и соединение с частью, находящейся под напряжением. Работодатель должен гарантировать, что, пока работник приближается, уходит или подключается к цепи, находящейся под напряжением, работник выдерживал минимальные расстояния приближения, установленные работодателем в соответствии с § 1926.960 (c) (1) (i), между работником и любыми заземленными частями, включая нижнюю стрелу и части грузовика, а также между работником и токопроводящими объектами, находящимися под напряжением с разным потенциалом.

(15) Размещение ковша возле проходного изолятора или изоляционной колонны. Пока ковш находится рядом с находящейся под напряжением втулкой или гирляндой изолятора, работодатель должен обеспечить соблюдение служащими минимальных расстояний между фазой и землей, установленных работодателем в соответствии с § 1926.960 (c) (1) (i), между всеми частями ведро и заземленный конец проходного изолятора или изоляционной колонны или любую другую заземленную поверхность.

(16) Handlines. Работодатель должен следить за тем, чтобы работники не пользовались ручками между ковшом и стрелой или между ковшом и землей. Однако сотрудники могут использовать непроводящие ручки от проводника до земли, если они не поддерживаются ковшом. Работодатель должен гарантировать, что никто не использует веревки, используемые для работы без экипажа на живой веревке, для других целей.

(17) Передача предметов сотруднику. Работодатель должен гарантировать, что сотрудники не пропускают неизолированное оборудование или материалы между столбом или конструкцией и канатным подъемником, в то время как работник, работающий с ковша, прикреплен к части, находящейся под напряжением.

(18) Непроводящий измерительный прибор. Непроводящее измерительное устройство должно быть легко доступно для сотрудников, выполняющих работу без экипажа под напряжением, чтобы помочь им поддерживать требуемое минимальное расстояние приближения.

(г) Башни и строения. Следующие требования применяются к работам, выполняемым на опорах или других конструкциях, поддерживающих воздушные линии.

(1) Работы под башнями и сооружениями. Работодатель должен гарантировать, что ни один сотрудник не находится под башней или строением во время работы, за исключением случаев, когда работодатель может продемонстрировать, что такое рабочее место необходимо для оказания помощи работникам, работающим выше.

(2) Строки тегов. Работодатель должен гарантировать, что сотрудники используют шнуры для ярлыков или другие аналогичные устройства для поддержания контроля над поднимаемыми или позиционируемыми секциями башни, если только работодатель не сможет продемонстрировать, что использование таких устройств создаст большую опасность для сотрудников.

(3) Отсоединение линий нагрузки. Работодатель должен гарантировать, что работники не отсоединят грузовую марку от элемента или секции, пока они не закрепят груз надежно.

(4) Неблагоприятные погодные условия.Работодатель должен гарантировать, что, за исключением процедур аварийного восстановления, сотрудники прекращают работу, когда неблагоприятные погодные условия могут сделать работу опасной, несмотря на методы работы, требуемые данным подразделом.

Примечание к пункту (d) (4):

Грозы в окрестностях, сильные ветры, снежные бури и ледяные бури являются примерами неблагоприятных погодных условий, которые делают эту работу слишком опасной для выполнения даже после того, как работодатель применяет методы работы, требуемые этим подразделом.

.

Добавить комментарий

Ваш адрес email не будет опубликован. Обязательные поля помечены *